Anda di halaman 1dari 450
STRENGTH OF MATERIALS Parr T Elementary Theory and Problems S. TIMOSHENKO Prefer of Thain and Roping Mechnien ‘iors meray SECOND EDITIONTRNTH PRINTING D. VAN NOSTRAND COMPANY, Inc. ‘ToRoNTO NEW YORK LONDON , Van Nostrand! Company, Ine, 250 Fourth Aven, New York 3 Yan Nostrand Company, (Cand), Ld, 228 Bor Stree, Tounto Macmillan & Company, 1d, St. Martin's Stree London, W.C. 2 eorrat. 109, 1840 oy D. VAN NOSTRAND COMPANY, Ine Thich, tan te, may nt ‘Manon rom he eran the punks, Prt Pua, May 1920 Reprint March 192, Fay 1998 Pabroary 1888 Second Kain, June 1940 Reprint alr 1 aly 048 onony 184 Av 144, Mey 1046 Moy 1045, Peiray 1057, Ang 1967 “Aor 1048 y PREFACE TO THE SECOND EDITION In preparing the second edition of this volume, an effort thas been made to adapt the book to the teaching requirements of our engineering schools. With this in view, a portion of the material of a more advanced character which was contained in the previous edi- tion of this volume has been removed and will be inchided in the new edition of the second volume. At the same time, some portions of the book, which were only briefly discussed in the fist edition, have been expanded with the intention of making the book easier to read for the beginner. For this reason, chapter IT, desling with combined stresses, has been centirely rewritten.” Also, the portion ofthe hook dealing with shearing force and bending moment diagrams has been ex- panded, and a considerable amount of material has been added to the discussion of deflection curves by the integration method. A diseussion of column theory and its application thas been included in chapter VILL, since this subject is usually required in undergraduate courses of strength of materials, Several additions have been made to chapter X dealing with the application of strain energy methods to the solution of statically indetermined problems. In various parts of the book there are many new problems which may be usefl for class and home work. ‘Several changes in the notations have been made to con- form to the requirements of American Standard Symbols for ‘Mechanics of Solid Bodies recently adopted by The American Society of Mechanical Engincers. Te is hoped that with the changes made the book will be found more satisfactory for teaching the undergraduate course of stength of materials and that it wil furnish a better foundation for the study of the more advanced material discussed in the second volume. S. Tiwosuenko joes ¥ PREFACE TO THE FIRST EDITION At the present time, a decided change is taking place in the attitude of designers towards the application of analytical ‘methods in the solution of engineering problems. Desig nnolonger based principally upon empitieal formulas. The im portance of analytical methods combined with laboratory ‘experiments in the solution of technical problems is beeoming senerally accepted. ‘Types of machines and structures are changing very rap- idly, especially in the new fields of industey, and ususlly tne does not permit the accumulation of the necessary empirical data, The size and cost of structures are constantly inereas- ing, which consequently creates a severe demand for greater reliability in structures. The economical factor in design tunder the present conditions of competition is becoming of rowing importance, ‘The construction must be sufficiently strong and reliable, and yet it must be designed with the Breatest possible saving in material. Under such conditions, the problem of a designer becomes extremely difficult. Re Guction in weight involves an increase in working stresses, “which can be safely allowed only on a basis of eareful analysis ‘of stress distribution inthe structure and experimental investi. Bation of the mechanical properties of the materials em: ployed. eis the aim of this book to present problems such that the student's attention will be focussed on the practical applica. tions of the subject. If ths is attained, and results, in some ‘measure, in increased correlation between the studies of strength of materials and engincering design, an important forward step will have been made. ‘The book is divided inco two volumes. The first volume ‘contains principally material which is usually covered. in sd courses of strength of materiale in our engineering “ PREFACE TO THE FIRST EDITION schools, The more advanced portions of the subject are of interest chiefly to graduate students and research engincer, and are incorporated in the second volume of the book. This contains also the new developments of practical imporeance in the field of strength of material. In writing the frst volume of strength of materials, atten- tion. was given to simplifying all derivations as much a5 possible so that a student with the usual preparation in math matics willbe able to readit without difficulty.» Forexample, in deriving the theory of the deflection curve, the area moment method was extensively used. In this manner, a considerable simplification was made in deriving the deflections of beams for various loading and supporting conditions. In discussing. statically indeterminate systems, the methad of superposition was applied, which proves very useful in treating such problems as continuous beams and frames. For explaining combined stresses and deriving principal stresses, use was made of the ‘Mobr’s circle, which represents a substantial simplification in the presentation ofthis portion of the theory. Using these methods of simplifying the presentation, the author was able to condense the material and to discuss some problems of a more advanced character. For example, in discussing torsion, the twist of rectangular bars and of rolled sections, such as angles, channels, and T beams, is considered, ‘The deformation and stress in helical springs are discussed in detail, In the theory of bending, the case of non-symmetrica cross sections is discussed, the cewer of twist is defined and ‘explained, and the effect of shearing force on the deflection of ‘beams is considered. ‘The gencral theory of the bending of bbeams, the materials of which do not follow Hooke’s lay is ‘given andis applied in the bending of beams beyond the yielding point. ‘The bending of reinforced concrete beams is given consideration. Tn discussing combinations of direc and bend- ing stress, the elect of deflections on the bending moment considered, and the limitation of the method of superposition is explained. In treating combined bending and torsion, the cases of rectangular and elliptical cross sections are dis. PREFACE TO THE FIRST EDITION vii cussed, and spplications in the design of crankshafis are given.’ Considerable space in the book is devoted to methods for solving elasticity problems based on the consideration of the strain energy of clastic bodies. These methods are ap- plied in discussing statically indeterminate systems, ‘The sercases produced by impact are also discussed. All these problems of a more advanced character are printed in small type, and may be omitted during the frst reading of che book. ‘The book is illustrated with a number of problems to which solutions are presented. In many eases, the problems are chosen so as to widen the field covered by the text and ¢o illustrate the aplication of the theory in the solution of design problems. Itis hoped that these problems wil be of interest for teaching purposes, and also useful for designers, The author takes this opporenity of thanking his friends who have asisted him by suggestions, reading of manuseripe and proofs, particularly Messrs, W.'M. Coates and Le Hi. Donnell, teachers of mathematics and mechanics in’ the Engineering College of the University of Michigan, and Mr. FL, Everett of the Department of Enginering Research of the University of Michigan. He is indebted also to Mr. F, C. Wilharm forthe preparation of drawings, to Mrs, ED. Webster for the typing of the manuscript, and to the Van Nostrand Company for its ear in the publication ofthe book. S. Trsosnexo, ‘Aun Anvos, Momcan May tins NOTATIONS: en em €x----Normal stresses on planes perpendicular to 3, 9 and z axes, [Normal stress on plane perpendicular to direction oer Normal stress at yield point. « [Normal working stress + Shearing stress ‘Tan Ton Tas Shearing stresses parallel tox, and 2 axes on the planes perpendicular to y, 2 and x axes, Working seress in shear ‘Total elongation, total deflection Unit elongation Unit elongations in x » and 2 directions Unie shear, weight per unit volume ‘Modulus of elasticity in tension and compression ‘Modulus of elasticity in shear Poisson's ratio ‘Volume expansion Modulus of elasticity of volume Torque Bending moment in a beam Shearing force in a beam Cross sectional area ‘Moments of inertia of a plane figure with respect toy and 2a Radi of gyration corresponding to Jy I. Polar moment of inertia Section modulus Torsional rigidity Length of a bar, span of beam Concentrated forces “Temperature, thickness f PANEERP Pome Sty Dros > SRpPSshes NOTATIONS Coefficient of thermal expansion, numerical eoef- ficient Strain energy Strain energy per unie volume Depth of a beam, thickness ofa plate Load per unit length Angles Pressure Diameters Radi Weight, load CONTENTS: 1. Testow avo Cowprestion winuis mux Elastic Lair lastcty | 2 Hooke's Law| 1. The Tensile Test Diagram f Working Stes §. Stess and Strain Produced in a Bar by its Own Weight 6, Statially Indeterminate Problems in Tension and Compression = = anne 7 beaded Tena Ses - Extension of 2 Circular Ring IL, Awauvsts ov Seuss ano Sraaie « 9. Variation of the Stress with the Orientation ofthe ‘Gross Section for Simple Tension and. Compres- 10, The Cirle of Sess 11, Tension or Compression in Directions “The Cirle of Sere for Combined Stresses) | Principal Stresses : ‘Analysis of Stain in the Case of Simple Tension Strxin in the Case of Tension of Compression it “Two Perpendicular Directions Pure Shear. Modulus in Shear Working Stresses in Shear ss. ‘Tension or Compression in ‘Three Perpendicular Diese iinet hence wo Pependiclar IIL Sueanivo Force ax Bexoine Moxent 15. Types of Beams 22. Rening Memes ant Shain Force 21; Relation Between Bending Moment and Sheasing ‘Force . : 12, Bending Moment and Shearing Force Diagrams © TV, Seansses iy Teansvensatty Loaogo Beas « 23, Pure Bending 3 3 “ Pia e ss 8 6 6 66 6 n is vI vu. Pa 3 z 2. 28. 29. ° CONTENTS Various Shapes of Cross Sections of Beams. General Case of Tranewerally Losded Beare Shearing Stress in Bending’ Distribution of Shearing Stcses in the Case of & Circular Cros Section Distribution of Shearing Steses in I Beams Principal Streses in Bending Streser in Buileup Beams Derzzorion oF Transveasatty Loaneo Beams a” 3 Be oe 35 6 3 ea ‘Dicrensal Equation ofthe Deflection Carve Bending of « Uniformly Loaded Beam Dafeston of @ Simply Supported Beam Loaded witha Concentrated Load Daermination of Defectons bythe wie’ of dhe ening Moment Diagrams Method of Super- poston « Detection of a Cantever eam by thé Area? Moment Metind vc es cs yer Daten at Sin Sopra‘ the ‘AracMoment Metwad Dafcton of Bears with Over ‘Phe Deflection of Bears When the Loads Are Not Farle o One of the Two Priya Planes of Bending gece of 8 ‘Beans Srarteatuy INoerenonvare Prosteusix BENDING. - 46. Redundant Constraines 4 Beam Buln t One Eid and Supported t he ‘Other » 442, Beam with Both Ends Bui in 3h Brames “ EE. Beams on Three Supports Continuous Beams sans oF Varian Ctoss Secrion. Beans or Two 6 rn * » Maseniats a Beams of Varatle Cross Section Beams of Two Different Materials Reinforeed-Concrete Beams» Shearing Stresses in Reinforced. Concrete Beams 4 4 7 1 ug 5 8 my a 2 ne ery CONTENTS ai VI. Covstsso Bexore aro Tension ot Coursssion "Turon ov Covvnes = . 236 so. Bending Accompanied by Compresion or Tenion | 228, $f, Becente Loading of «Shore Strat ae $2 The Core of Section LL 5, Becentie Compression ofaSlender Column <<< 239 i Col Lal ee s7- Empitical Formulas for Golarsa Design < a8 IX, Torstox avo Connie Brwoixe axo Tosston 261 48, Torsion of « Circular Shaft Tit ast & Tee of alow Shaft 1 a8 ‘The Shafe of Rectangelar Crote Section | <1 S26 64, Helical Spring, Close Called i 62. Combined Bending and Twist in X. Exanor or Sra 6. Elastic Strain Energy in Tension Gq Tension Produced by Impact. 6. Elie Sun Eneray nha and Fis [Esti Strain Energy in Bending ae fp, Bending Produced by Impact == Ss ss 68; The Geteral Expression for Strain Energy = 65. The Theorem of Carighano 7: Appliation of Castilno ‘Theorem in Solon of ‘Seaticaly Indeterminate Problems 30 17h. The Reciprocal Theorem DIIIIT by FP. Exceptional Cases SS tM ‘Mowers or Inearis oF Puaxe Fiounes . - «<3 1. The Moment of Inertia of Place Area with Respect to ‘tn Ans in Ts Plane : a8 U. Polar Moment of Inertia ofa Plane Area 1211 as NE Transfer of Asis os ur IV. Product of Inertia, Principal Aves 'V. Change of Direction of Axis. Determination of the Principal Axes «= ss Avmion NOEX s+ eee 26 38S suneer DER ee a +397 STRENGTH OF MATERIALS PART J CHAPTER 1 ENGION AND COMPRESSION WITHIN THE ELASTIC LiMIT 1, Blasticity.—We assunie that a body consists of small particles, or molecules, between which forees are acting. ‘These molecular forces resist the change in the form of the body which external forces tend to produce. If such external forces are applied to the body, its particles are displaced and the matual displacements continue untl equilibrium is estab- lished between the external and internal forces. It is said in such a case that the body is in a state of sirain. During deformation the external forces acting upon the body do work, and this work is transformed completely or partially into the potential energy of strain. An example of such an sccumulation of potential energy in a strained body is the case of a watch spring. If the forces which produced the deformation of the body are now gradually diminished, the body returns ‘wholly or partly tots iil shape and dure | ing this reversed deformation the potendal | Pe nergy of strain, accumulated in the body, |4]'T may be recovered in the form of external | [I work ie Take, for instance, a prismatical bar loaded at the end as’ shown in Fig. 1. Under the action of this load a certain clon gation of the bar will take place. ‘The point of application ‘of the load will then move in a downward direction and Positive work will be done by the load during this motion. Prat 2 STRENGTH OF MATERIALS ‘When the load is diminished, the elongation of the bar dimin- ishes also, the loaded end of the bar moves up and the poten tial energy of strain will be transformed into the work of moving the load in the upward direction. ‘The property of bodies of returning, after unloading, to their initial form is called elasticity. Te is said that the body is perfectly elaitic if it recovers its original shape completely after unloading; it is partially clastic if the deformation, produced by the external forces, does not disappear com: pletely after unloading. In the case of a perfectly elastic body the work done by the external forces during deformation will be completely transformed into the potential energy of strain, In the case of a partially elastic body, part of the work done by the external forces during deformation will be dlssipated in the form of heat, which will be developed in the body during the non-elastie deformation. Experiments show that such sttuctural materials as steel, wood and stone may bbe considered as perfectly elastic within certain Timits, which depend upon the properties of the material. Assuming that the external forces acting upon the structure are known, itis ‘8 fundamental problem for the designer eo establish such proportions of the members of the structure that it will approach the condition of a perfectly elastic body under all service conditions. Only under such conditions will we have continued reliable service from the structure and no permanent set in ies members. 2. Hooke's Law.—By direct experiment with the exten- sion of prismatical bars (Fig. 1) it has been established for ‘many structural materials chat within certain limits the elon- gation of the bar is proportional to the tensile foree. This simple linear relationship between the foree and the elon tion which it produces was first formulated by the En scientist Robert Hooke in 1678 and bears his name. Using the notation P= force producing extension of bar, 1 = length of bar "Robert Hooks, De Parents restatva, London, 178 TENSION AND COMPRESSION 5 A = cross sectional area of bar, total elongation of bar, ‘elastic conatane of the material, called its Mod dus of Elasticity, Hooke's experimental law may be given by the following equation PL a3 @ The elongation of the bar is proportional tothe tensile force and to the length of the bar and inversely proportional to the eross sectional area and to the modulus of elasticity. In making tensile tests precautions are usually taken to scare central application of the tensile foree. In this manner any bending of the bar will be prevented. Excluding from coo sideration those portions of the bar in the vicinity of the applied forces? it may be assumed that daring tension all longitudinal fibers of the prismatical bar have the same jon and che cross sections of the bar originally plane and perpendicular to the axis of the bar remain so after exten “Tn discussing the magnitude of internal forces let us im= agine the bar cut into two parts by a cross section my and let us consider the equilibrium of the lower portion of the bar Fig. 1, J). At the lower end of this portion the tensile force P'is applied. On the upper end there are acting the forces representing the action of the particles of the upper portion of the strained bar on the particles of the lower Portion. ‘These forces are continuously distributed over the «ross section. A familiar example of such a continuous di. tribution of forces over a surface is that of « hydrostatic Pressure or of a steam pressure. In handling such contina- ously distributed forces the intensity of force, ie. the force per unit area, is of @ great importance. Im our case of axial tension, in which all fibers have the same elongation, the ste moe compcaad str dibton near che points of 200 ton of the cer Wi be dace intern Pare PoP 4 STRENGTH OF MATERIALS tribution of forces over the cross section my will be wis farm. Taking into account that the sum of these forces, from the condition of equilibrium (Fig. 1, 6), must be equal to P and denoting the force per unit of cross sectional area by o, we obtain ® ‘This force per unit area is called stress. In the following, the force will be measured in pounds and the area in square hes so that the stress will he measured in pounds per square inch. The elongation of the bar per unit lengeh is deter- mined by the equation 5 ® and is called the unit clongaion or the tensile sain. Using fs. (2) and (), Hook's law may be represented in the following form? @ and the unit elongation is easily calculated provided the stress and the modulus of elasticity of the material are known. The ‘unit elongation « is @ pure number representing the ratio of two lengths (see eq. 3); therefore, from eq. (4), it may be concluded that the modulus of elasticity is to be measured in the same units asthe stress ¢ ie, in pounds per square inch. In Table I, which follows, the average values of the modulus E for several materials are given in the first column Equations (1)~(4) may be used also in the case of the com- pression of prismatical bars. Then 8 will denote the total Tongitudinal contraction, « the compressive strain and the compressive stress. ‘The modulus of elasticity for compression is for mast structural materials the same as for tension. In calculations, tensile stress and tensile strain are considered ‘i positive, and compressive stress and strain a9 negative. 5 Bore dete on the mechavial properties of materials are given Ia Pare TENSION AND COMPRESSION 5 Maceameat Phoremis or Mares Yiu Prine | imate Sungsh wea Teint Ton ce ae sex tx rte x 12 |e 16s x eked 3 thi | 3p tt| ge tah te | tons Desk, Toit] Sp hae to Fe thes Ho Capper od aie xt ose ome kta sexe Fei de pia ts Pt x Gina npn |g 1 peo ‘Problems 1, Determine the total elongation ofa see bar 25. long, i the tensile stress is equal to 15 % to Ibs. per ain, “Arwen eex re SE Zin aexras 2, Determine the tensile force on eylindsial stel bar of one inch diameter, ifthe une elongation is equal to 7 % 10" ‘Solution. "The tensile stress in the bar, Hot Cis ©F = 21 X 104 Ths per sin ‘The tensile force, fom eg. (2) is Pe A= 21 X 108 x = 16,500 4. What i the ratio of the moduli of elasticity ofthe materiale ‘fro bats ofthe same nse ander the action of eal ene oees the nit elongatios ofthe bare aren the ratio 1/8. Determine these lengttons if one ofthe bars of sel the okt of copper and ‘he tensile stray i 10,00 Ib. por nc, Selation. ‘The modal are nveteiy proportional to the wait ‘elongations. For steel * = 19000 Bx” See" for copper "Tee 6 STRENGTH OF MATERIALS 4A prlsmatial steel bar 2 in. Jong i elongated /49 in, under ‘the Action of a tenile force. Find the magnitude of the force if ‘the volume of the bar is 35 in? §-A piece of wire 10 f. long subjected to tense force P 1,200 le, longates by Vin. Find the modslus of elasticity of ‘the material if the eros sectional ares of the wie is 04 i 3. The Tensile Test Diagram.—The proportionality be- ‘ween the tensile force and the elongation holds only up to 2 certain limiting value of the tensile stress, called the limit of proportionality, which depends upon the properties of the material, Beyond this limit, the relationship between the . "Determine thee Gros sctional aes ef the ba tnd th defection of he Pint when P = 00 Ib ee tooo Tere tnd eal ge ‘tiation ef the bas #3 Salon. From Fig. 4 (representing the conven for ek 10 STRENGTH OF MATERIALS librium ofthe hinge B, the tensile force in the bars is ‘The defection BB wil be found from the small right eviangle BB, in which the are BD, of radius equal tothe length ofthe bars, is considered as a perpendicular dropped upon 4B, which is the position of the bar /B after deformation, ‘Then the elongation of the bar 28 ie 10409 X18 X19 gin a pe ra tis seen thatthe change ofthe ange due tothe defection BB, is very small and the previous calculation of S, Based upon the assumption that 0 Jos is accurate enough. ‘3 Determine the total clngstion of the stcl har 4B having actos sectional area d = Yin? and submitted t0 the action of forces = 10,000 Tbs. tnd P = sce ibs. Fig 3), Solution. The tense force in the upper and loner portions of the bar is equal to Q and that inthe mide portion iv QP. Then the ctl XL He songation will ail t set0in L al 5900 X10 q Fxg xd i567 4. Determine'the cross setional dimensions of the wooden beam BC and of the steel bar 4 of the structure ABC, loaded at, when the working stress for wood is taken as.eq = 160, Ibs per sqrin. and for steel ev = 10,00 Ibs. per sq in. The load TENSION AND COMPRESSION " PP = 6909 ths The dimensions of the sectre are shown in Fig. 6°°Desernine the wert od che horoonal capenents of SE diane they aro tomo oh a i 1 Seen rom Pgs O) ging econo binge B similar tothe rangle 4BC of Eg. 8 (e), web pa Pa 9 librium of tooo Ibe 4j The cross sectional areas ofthe steel bar and ofthe wooden beam are an 5 19000. 22 in fe 1900 ne 168 7 ‘The total elongation ofthe steel bar and the total compression of the wooden eam sre sere io X12 13 15X10 = cots in, ‘To determine che displacement of the hinge B, due to deformation, ses are drawn with centers 4 and G (Fig. 6 a) and eal equal 0 the lengths of the elongated bar and of the compressed beam * spectively “Tey. inthe new pontion ofthe hinge 8 is shown on a larger scale in Fig. 60), where BB, is the elon: 2 STRENGTH OF MATERIALS tation of the steel bar and JB the compression ofthe wooden bear. ‘The dotted perpendicslars replace the arcs mentioned. above, ‘Then BB is che placement ofthe hinge B. The components of this dsplacemene may be easly obtained from the Bgure. ‘5: Determine in the previous problem the inclination of the ‘bar 2B to make its weight « minimum. Solution. If 6 denotes the angle between the bar and the horizontal beam and the lenge of the beam, then the length of the baris = hileos @ the tensile force inthe bar is 8 = PJsin Band the necesary cross sectional ares iv f= Plewsin 8, The volume of the bar wil be Tis seen that the volame and the weight of the bar beome ima nen tan as ra The square fame 4BCD (Figs 7, 2) consitng of vest bars oft neces sacioal area slated tothe tion of two force P ~ sozoo lin. n the direction of the diagonal. Determine the changes of the angls at--and C doe to deformation of the Frame Determine the changes of the came angles if the forces are flied a shown in ig. 7. Solution. In the ease shown in Fig. 7 (a) the diagonal will take the complete load P. Assuming that the hinge D and the Girection of the diagonal are stationary, the displacement of the hinge Bin the direction of the diagonal wil be equa w the elongation ‘of the diagonal 8 = PI/1E.- The determination of the new position of she hinge Cis indicated in the figure by dotted lines. Ieis cen from the small right triangle CC\C" that CC" = afya. Then the angle of rotation of the bar DC due to deformation of the frame sequal to TENSION AND COMPRESSION 8 jas Taian “Then the increase ofthe angle at Cwill be aaeian, ‘The so of the pole shows in 7 (set fo the sadn “pietermine the psion of the lad Pom the beam BD 3 that ihe force in the bar BC becomes 4 mati. Determine the tangle to make the volume ofthe bar BC a aintoom (Eg 8) Bat se APET Ph fons hr 2 come maxima ene ateie pion on the right st point. valine of the bar wil be minim when agen 1, Determine the neccary ero sctinal ate of he steel bar 20 (igo) ithe working sues on = "5p Tb. peso and the ceiieaiP Geeltced Seced” Se retin nd the loud per foot of the bam it By cote anioen d= 06 sin. 9: Determine the Mensary at sectional areas of the bars GB and Bo of the saucers nso 12009 6 teem in Fer 10 and WE nswer. Inthe ese. of srvctre To () the cots ta Sal so ode 9s Sin ant of the bar BC2.0 nga he cae of Fig. 10 econ seta an of te bal J shal og Seat the bare ay to. Sle prob jamming sa the miei ralainn and tae P='p = sogsolte pean “ STRENGTH OF MATERIALS 11, Find the comectona are ofthe bar CD in Fis toa and'1oB tnd heal gon of hse ba te natal ictal and y= Sons he. pero aS pole uring the hoe sappy one Jt of Bopper Chord nt inne fe fom pre ‘5. Stess and Strain Produced in a Bar by its Own Weight-In discussing the extension of a bar, Fig. only the load P applied atthe end nan taken into conse on, Ir the length of the bari large, x own weight nay produce Considerable additional stem and shosld be taken int ae- counts In ths ease the mania steel bea the bln tpper crs acetone Denoting by 7 the weight per anit telumeof the bar the complete weight wil bey andthe Iresimm sess Will be gion by the eon Pal P Pt at © “The second term onthe right side of eq (6) represents the sures produced by the weight of the bar. The weght of that portion of the bar Below's coms section at itance = from the lower end (Fg 1) sand the sree wl be given bythe equation Pape © Substituting the working stress oy for ome in ea. (6), the ‘equation for calculating the safe cross sectional area will be 4eata o Teis interesting to note that with inressing length! the bar's fown weight becomes more and more important, the deno nator ofthe right sie of en. (2) diiniohes and the necessary ross sectional area increases, When 21 = ou hey the Streas due to the weght of the bar alone Becomes equal fo the working stes, the right sie of eq. (8) becomes infite,”" Under sach circumstances iis imponsible to use 4 Prismatcal design and recourse to a bar of variable cross fection fs made. TENSION AND COMPRESSION 1s In calculating the total elongation of a prismatical bar submitted to the action of a tensile force P at the end and is own weigh, let us consider first the elongation of en ele- ment of length dx cut from the bar by two adjacent cross sections (see Fig. 1). Te may be assumed that along the very short length di the tensile stres is constant and is given by eq (7). Then the elongation dB of the element will be The total elongation of the bar will be obtained by summing. the elongations of all the elements. Then “Pie dow yt bo [Ea Retin. Comparing this with a. (1) tis cen tha the total elongation produced by the bar's own weight is equal to that produced by a load of half its weight applied at its end. Problems 1. Determine the eros sectional area of a vertical prismatic sees eg on lowe end ain Poco ln he 19.720 fet the woking sts = Too ba por eight of cable fot of see yobs. Determine al area, from eq. (8) is ye.800 4 = sarin we XT rope AXE wend Deering the conan of coil bar under th action iow welgh (Fig 1) the length of the bar the dameter ‘of the base fad an the weight per unit welume ofthe material 16 STRENGTH OF MATERIALS Soluion. The weight ofthe bar wil be th, real or any cross section at dintance «from the lower Cf eb the tense forces eal tote weight Sf the ker portion ofthe ba * oe Assuming that the temile force is uniformly distributed over the trons section * and considering the element of lengeh de as pri ‘matical bu, che elongation of this clemene wil be a= and he wl dng of he i piace el nie “This elongation is one third that of peismatical Bar of the same length (oe 9) 3. The vera prismatic ed of mine pomp iemoved up and down by a rane shat (lg 3) ‘Assuming tha the material tel and the werking resin ex = pon Ibs. pera ny determine the oss sectional ten of the rod ifthe restance of the piston daring motion downward is 0 Tbe nd = dori motion upard i 2,000 The. The length of fin the sed io go ct. Deter the necessary 1 length of th radiey of the crank f the stake of Fm, thepumplscqul cod in ‘elton. "The necesary cos sectional are of the rod willbe found fom eq, () by substituting P= 2,0 bs Then Spas 790 = oag8 int “The diference in rota elongation ofthe tod when it moves up and. ‘when it moves down is duet the restance ofthe pista and wil be mall Za asumpton is usifable when the angle of the cone TENSION AND COMPRESSION ” equl © = (20004 20):300018 ig ous X Og 7 “The radius ofthe crank should be SHEEN Sg aein 4 Lengths of wire of stel and aluminum ace suspended verti cally. Determine foreach the lengch at which the stres die tothe weight of the wire equals the ultimate strength if for steel wire Te 300}900 Ib. per sn andy = 490 lbs. pe cubic fot, and for ‘laminar wire ex = sooo Ibs. per‘. im andy = #70 Ibo. pee cn foo. Vdnomer. For steel = 88,00 fy for aluminum 1= 42900 ft Sin what proportion wil the maximum stress protean 4 prismatial barby te own wight incest fall the Gimensions of rebar ae increased in the proportion vs (Fig. 1)? dnswer The stress wil ncoase the rate a Norge pla consisting of wa piematin portions of equal length (Vig 13) i londed at the upper end by 2 compresive force P= Goozn lhe. Determine the touane of rasonry if he height ‘ofthe pili 120 fe its weight per cubic foot is 100 Ibs, and the {Sotpare evlume ith that ofa singh mae pal deine forthe same conditon Degas e ae ie mK Pe 1 Re 7. Solve the preceding problem assuming three prismatic pordons of equal length, 8. Determine the form of the pillar in Fig. 14 such that the stress in each cross section is just equal to 2x. The form satisfying this condition sealed the frm of egal srenth. Solution.” Considering 2 diferential clement, shaded in che 18 STRENGTH OF MATERIALS figure, i is evident thatthe compressive force on the erots section aman is larger than that on the cross action mn by the magnitude cof the weight of the clement. Thos since the stress in both cross sections i tobe the same and equal to auy the diflerence df inthe ‘rosssectional area mast be such as 9 compensate forthe diference inthe compressive force. Hence Aes = dd @ here the right side of the equation represents the weight of the clement. Dividing this equation by es nd integrating we Sind Sa-Se log 4G from which and A= Conn, o where « isthe hase of natural logarithms and @-= e®, At x this equation gives for the eres-sectional area atthe top of the pill Dow = ¢. But the crosssectional area at the top is equal to Plous hence C= Plas and equation (2) becomes Aa Een, © ‘The crone-sectional area at the bottom of the pillar is obtained by substituting « = Zin equation () which gives > Anse os @ 49. Find the volume of the masonry fora yillar of equal strength desnad fone hc onion of eg Salon. ‘By tsing equation (i) the derence ofthe cose sectional arcasat the botom of the ilar andar steps found be ‘This diference multiplied by the working stress ex evidently gives the weight ofthe pila; its volume is thas 1 = eta) = goatee TENSION AND COMPRESSION 9 6. Statically Indeterminate Problems in Tension and Compression—There are cases in which the axial forces acting in the bars of a structure cannot be determined from the equations of statics alone and the deformation of the structure must be taken into consideration, Such structures are called statically indeterminate systems. A simple example of such a system is shown in Fig. 15, ‘The load P produces extension in the bars OB, OC and OD, which are in the same plane. ‘The conditions for equilibrium of the hinge O give ewo equa- tions of statics which are not sufficient to determine the three tunknown tensile forces in the bars, and for a third equation a consideration ofthe deformation of the system becomes neces- sary. Let us assume, for plicity, that the system is sym- metrical with respect to the vertical axis OC, that the ver- Pep tical bar is of steel with 4, and E, as the cross sectional area and the modulus of elasticity for the material, and that the inlined bars are of copper with 4, and E-as area and modulus. The length of the vertical bar is /and that of the inclined bars is Jcos a. Denoting by X the tensile force in the vertical bar and by ¥ the forces in the inclined bars, the only equation of equilibrium for the hinge O in this case of symmetry will be X42¥coa= 7, @ In order to derive the second equation necessary for deter- 1g the unknown quantities X and Y, the deformed con. figuration of the system indicated in the figure by dotted lines must be considered. Let 8 be the total elongation of the vertical bar under the action of the losd P; then the clongation’&, of the inclined bars will be found from the ‘wiangle OFO,. Assuming that these elongations are: very » STRENGTH OF MATERIALS small, the circular are OF from the center D may be replaced by a perpendicular line and the angle at O, may be taken ‘qual to the initial angle «then B= Beosa. ‘The unit elongations and the stresses for the vertical and the inclined bars will be 3 Es Beota | _ Bb costa wb and a tS, BEE, respectively. Then the forces in the bars will be obtained ‘by multiplying the stresses by the cross sectional areas as follows: Xae from which 4A aot, SEB, ¥ = Xeota $e Substituting in ¢q, (4), we obtain x 7 ro) iy aa 1+ 2 08 a Se Its seen that the force X depends not only upon the angle of inclination «but also upon the cross sectional areas and the mechanical properties of che materials of the bars, In the particular case in which all bars have the same cross section and the same modulus we obtain, from €q. (10), P X= prara When approaches 2er, cosa approaches unity, and the force in the vertical bor approaches 1/32. When « ap proaches 96% the inclined bars become very long and the Eomplete load will be taken by the middle bar. {As another example of statically indeterminate ayotem Jee us consider a prismaticel bar with builein ends, loaded ‘TENSION AND COMPRESSION a axially at an intermediate cross section mm (Pig, 16). ‘The load P will be in equilibrium with the reactions R and Ri at the ends and we have P=R+R. o In order to derive the second equation for determining the forces R and R, the deformation of the bar must be consid. cred. ‘The load P with the force R produces shortening of the lower portion of the bar and with the force R, clong of the upper portion. ‘The total shortening of one partis eal to the tol elongation of he eA ther. ‘Then, by wing eq (y weobeain, Raa _ Rb i aE" : Hence zm, @ a the forces R and Ry are inversely propor. tional to the distances of their points of appli cation from the loaded cross section mn. Now from eqs. (0) and (d) the magnitudes of these forces and the stresses in the bar may be readily calculated, Problems 1, A ste cylinder and x copper tube are compresed between the pte oa press (Figs 17). Determine the stcses tel aad iti 22 fe se eee tlt a nw siti Wateer acta — siete ee mn Euneisae capone ane kat Bee maine afl est ce Sah Bi Res mn d 2 STRENGTH OF MATERIALS is os aS TH tee Phe Erato Substituting numerical values, we obtain = Ecc = 30h Ibe pr ag ns ee = 1690 lbs. per sq. in unit compression c= et x 108 2 Acoluma of reinforced concrete come prescd by « force P= ooo Ibe. What part ofthis load wl be enemy the conerete Sd what pat by the ste if the eos sectional rea of the ste) is only 1/to of the rose ‘etional ares ofthe concrete? “yA rigid body 4B of weight Q hangs on thee vertical wites symmetrically situated Feit, with respect to the center of gravity C of the body (Big. 18). Determine the tensile forces inthe wires ifthe mle wie is of stecl andthe two others of copper.” Cros sectional areas ofall wines are eu. “Suggestion. Use method of problem 4. Bins he res nar et Y of siguate table, Big. ty prodoced By ‘he loa P acting on one agonal The °-S=a— top of the tale andthe or are as = ‘ned absolutely sgid andthe legs are “FP ached to the Mor that hey fan ‘undergo tension as well as compression : ‘Slaon. Assuming. that the new . position of the top of the table is that 7 Indicated by the dotted. line mn, the compresion of legs 2nd 4 will be the i Sverage of that of legs tant 3. Hence a¥=X4Z be | and since 2¥ + Xf Z = P we obtain Poe op a¥=X4Z= 40. cy) ‘An additional equation for determining X and Z is obtained by taking the momene ofall Uke forces with rexpert to the horizontal ‘xis 0 ~ O parallel to y and inthe plane ofthe force P. Then Xavi +o) + AP re = Zavt ~ 0, o TENSION AND COMPRESSION 2 From (2) and (8) we obtain es p ‘ er(j-sa) 2s an a(trs When ¢ > ly, X becomes negative. This indiates that there sa be resion ag Determine the cs inthe leg ofthe above table when the loads applied a the pint withthe coainats Hint. Yn solving thi problem ie should be note that when the point of application ofthe load Pis maton the diagonal oft {abl this loud may be replaced by two nds stacey efureon the oad # and applied at pnts onthe we diagnale "The fos roducedin the eu by each of these twolonds ure found asetplsiead Bove Sumaing he fet ofthe ro com vent loads, the foes nthe legs for any post fon of the lad P may be founda? A rectangular frame with diagonals is fubmitted tothe action of compressive res P (Fig! 20. "Determine the forces in the bars they ate all of the same material, the cron see tional area ofthe verdes, and that ofthe remaining bars. “elton. Let X be the compresive free in cach verticah ¥ the compremive fore a cael aga hd 2th ene foe nach horizontal bar. Then from the condition of cqulibium of one ofthe hinges Y= shinalP—X); 2 Yeora (P~ xX) cota (@) ‘The third equation will be obtained from the condition that the sc after deformation remaine rectangular by sieuc of symmetry; fore vy from this, neglecting the small quantities of higher order, we gee esr ex oz a aE AE o Pere rere e+ u STRENGTH OF MATERIALS Solving eqs. (2) and (3), the following value ofthe forceina diagonal willbe obeaned Pp Yost a, a PF 48 Fcwatsina "The forges in other bars will now be easly determined from eqs. a) "7 Salve the above problem, assuming «= hy f= 5 and P= sooo Ibs 4, What stresies will be produced in a steel bolt and a copper tube (Fig. 21) by f of a turn of the ‘nut if the length of the bole /= jo in the pitch of the bole thread A= in, the area of the cross see- tion ofthe bolt, = 1sqc inch, the areaof the cross section of the tube = 2 sq inches? Solution. Let X denote the unknown tensile force in the bolt and the compresive force ia the tube. * ‘The magnitude of X willbe found from the condition ‘hat the extension of the bolt plus the shortening of the tube is equal tothe displacement of the nut along the bole. In ur cate, assuming the length ofthe tube equal tothe Jengeh of the bol, we obtain 16100 Ibs. per 9. in The tensile stress in the oleis = Xi, “aid = Bago Ibs per ‘The compressive stzes in the tbe is 9, What change in the stresses calculated in che above problem wil be produced by tensile forces P= 500 lbs. applied the The ofthe bole? ‘Solution. Let X denote the increase in the tensile force in the bole and ¥ the decrease jn the compressive force in the tube Then from the condition of equilibrium, X+Y=P. © ‘A second equation may be writtzn down from the consideration thatthe nit elongation of the bolt and tube under the application TENSION AND COMPRESSION 2s ofthe forces P must be equals Xe AE,” HE. o From eas, (a) and (8) the forces X and ¥ and the scomesponding stresses af easly calculated Je. A posnatal iar wih ble end onde silly at two intermediate cos section Fig. 2 feree P.and Pu. Determine the reactons aad Re Un, "Slatin wl betaine by singe.) on pge 3, calculating the eactns produced Up cag Determine the reactions when 47094 bm og and P= aPy= 1coobs 11. Determine the forces in the bars ofthe 19 system, shown ia Fig. ay where Oo isan au of Symmes sr, ‘The tn fen the ba O8 sail othe compesrive force in the bar fndis Pissing. he force inthe horoneal bar Spill nit a. 13, Solve problem 1o suming that the lower portion of length «ofthe bar has a cront-ectinal arta to time ger than eet sectional atea ofthe two upper pare of eng and 7. Initial and Thermal Stresses.—In a statically indeter- minate system it is posible to have some inital stresses Produced in assembly and due to inaccuracies in the lengehs of the bars or to intentional deviations from the correct values of these lengths. These stresses will exist when external loads are absent, and depend only upon the geometrical pro- Portions of the system, on the mechanical properties of the eaters and onthe magnitde ofthe insecure. Asin or example, that the system represented in Fig. 15 has, by. mistake, + 0 asthe length of the vertical bar instead of Then after asembling the bars BO and DO, the vertical bat can be put into place only after initial compression and due ‘© this fact it will produce some tensile fore in the inclined bars, Let X denote the compressive force in the vertical bar, which finally takes place after asembly. “Then the ‘corresponding tensile frcein the inclined bars will be Xs cos 4 pede + Pra a8 6 STRENGTH OF MATERIALS and the displacement of the hinge O due to the extension of these bars will be (see eq. J, p. 20) © ‘The shortening of the vertical bar will be XL ® From elementary geometrical considerations, the displace rent of the hinge O, together with the shortening of the ‘vertical bar, must be equal to the error ain the length ofthe vertical bar. This gives the following equation for deter- mining X: a xi Xx Ta carat Hence tte ay (+ ee) [Now the initial stresses in all the bare may be calculated. Expansion ofthe bars of a system due to changes in tempera- ture may have also the same effect as inaccuracies in lengths. ‘Assume a bar with built-in ends. If the temperature of the bar is raised from fy to # and thermal expansion is prevented by the reactions at the ends, there will be produced in the bar compressive stresses, whose magnitude may be caleulated from the condition that the length remains unchanged. Let 4 denote the coeficient of thermal expansion and ¢ the com- pressive stress produced by the reactions. Then the equation for determining o will be eve from which o= Bat 10 os Asa second example, let us consider the sytem represented in Figg and sone thatthe vera! bur 8 honed fom TENSION AND COMPRESSION 2 the assembly temperature % to a new temperature & ‘The corresponding thermal expansion will be partially prevented by the two other bars of the system, and certain compressive ll develop in the vertical bar and tensile stresses lined bars. The magnitude of the compressive force in the vertical bar will be given by eq. (11), in which instead of the magnitude @ of the inaccuracy in length we substitute the thermal expansion al{? — 4) of the vertical bar. Problems 1, Ther of tramway ae weed together at $0 Falreni ‘What stresses will be produced in these rails. ‘wher heated| by the nto 10s ifthe cockcnt of thermal epanton ofl yo to fasmey egos te pani 2. What change of serene willbe produced inthe case repre. sented in Fig 21 by increasing the tnperatre bom ty be fPeicint of expansion of stele and that of eoppet ak Solin, Baw t the fc ha o> ete neg tempera sare produces compreon inthe copper and tenon the ech. ‘The une lngation of the coppers of te stl sl be ea Denoting by 5 the ineran in he tn force nthe ble ue the change of tmpertre ne obtain x aut = 19 + Fe att from which ‘he change inthe stress in the bolt and inthe tube may be Caleaiatednow inthe ual way 2A stip of copper i soldered rms, been two wipe of ate (Eg 39) {ke Whar snes wil be produced fa the sland in the coppe by ae the temperate ofthe ate fom oF s The same method’ asin the ugetion, The same mthed a in the previous problem ‘should be used. Pe i “Wht sree wil be produced in the bare of the system 28 STRENGTH OF MATERIALS represented in Fig. 15 i the temperatare ofall the bars be raised Fométo? lon. Let X denote the temsile force produced in the ste! fan increase in temperature. Then from the condition of ileum of the hinge Oe canbe seen then the copper bare compressive. forces act, equal to X/s-cosas_consequemtly the ‘ongaton of the sts bar Becomes snd the elongation of the copper buts is LM aa” Tet aE A= ale) Furthermore from previous consideration (se p. 20), B= dona; therfore aul sa + from which thse Toorade, “The streies in the stel and in the copper will now be obtained fom the following equations: a* x as Feed 5 Assuming cha inthe case shown in Fig. 17a constant load P= scojoe ie applied at an inital temperature fy determine at that increase intemperate the load will be completly transmitted tthe copper ifax = 9 X lo" and a= 92 1o™ Solution mai | tats >= FE f= tym 75.4 degrees Fahrenheit -ae- a= from which TENSION AND COMPRESSION » 6. A steel bar consisting of two portions of lengths‘, and f and cross-sectional areas dy and vis xed atthe ends.” Find the ‘thermal sresses ifthe temperature rises by 109 degrees Fabreaheit (Resume A= fy du Ady and an = 70% 10 "7 Find the thermal stress in the system shown in Fig. 24 if the temperature of ll these stripe rises by 100 degrece Fahrent helt. The thickness of each of the three step is the same and the cooficients of thermal’ expansion are ay — 70 X 10" and * acm ga X to Assume E, The temperature of the system shown in Fig. 16 rss by too ogres Falenhelt. Find the thermal stresses if al three bars are ‘ofsteel and have equal cross-sectional areas. "Take a, — 70% To? ZX ih pe ind the stresses in the wires of the system shown in Fig. 18 ifehecronactonal are ofthe wires. sq7iny the load = 9.00 Fhe, and che temperature of the system rises alter assembly by 19 degrees Fahrenhere To. Determine the stresses which wil be built up in the syters ‘represented in Fig. 20 if the temperature of the upper horizontal bar ses fom fs ¢0# degrees 8, Extension of a Circular Ring—If uniformly distributed radial forces act along the circumference of a thin circular ving (Fig. 25), uniform enlargement of the ring will he pro- © Fe 4 duced. In order to determine the tensile force P in the ring let us imagine that the ring is cut atthe horizontal diametral section (Fig. 25, 8) and consider the upper portion as a free body. If g denotes the uniform load per unit length of the x0 STRENGTH OF MATERIALS centerline of the ring and r is the radius of the center line, the force acting on an element of the ring cut out by two. adjacent cross sections will be qrde, where do is the central angle, corresponding to the element. Taking the sum of the vertical components of all the forees acting on the half ring, the following equation of equilibrium will be obtained: SC ersin ode = 207, from w Poon 3) ‘The tensile stress in che ring will now be obtained by dividing the force P by the eross sectional area of the ring. In practical applications very often the determination of tensile stresses in a rotating ring is necessary. Then g repre- sents the centrifugal force per unit length of the ring and is given by the equation: (a) =F, in which w is the weight of the ring per unit length, ris the radius of the center line, 9 is the velocity of the ring at the radius 7, and g is aceleration due to gravity. Substiuting this expression for gin eg. (13), we obtain p=, £ land the corresponding tensile stress will be Put _ yt ; 0-5-4 8. i) Tes seen that the stress is proportional to the density vig of the material and tothe square of the peripheral velocity. F 1a steel ring and for the velocity » = 109 feet per second this stress becomes 1,060 Ibs. per sq. in. Then forthe same mate rial and for any other velocity othe stress will be 0.106 X ot in Ibs. per sq in, when vis in feet per sec. TENSION AND COMPRESSION a Problems _ Desert ele ain the clin wal of the pres shown in Fig. yf che inner diameter is fo ine and the thick Bes ofthe wall vIn “Slition. "The maximums hydrostatic pressure, wil be found fom the equation ee 1. Determine the tens the cylinder from hich p= 1270Ibe.persg-in. Cutting out fom the cinder an elemental rng of width tin in the direction of the ar of she eplinder and using (13) in whic, for this se, = p= 1290 Ib perine andr 5 ins, we obtain ap x oa Fx AS = 6350s, pr sgn 2. A copper tube is fitted over stel tube at a high temperature 4 (Big. 26, the it being such that no pressure eat betwee eas A thie temperature.» Determine the stones which willbe produce inthe copper and inthe feel when coded to room temperature ifthe ter Gameter of the sel tube in) the thik: Sep of heel ae and that ef the copes gliiton. Due tothe diference in the ox ficients of expansion ac and ay thre will bea . Exzrare eewcen the ‘outer and the inner tubes: ae fir cooling. Let * denote the pressure per square inch; thea the tensile stress in the copper tube will be a ok land the compressive stress inthe steel ‘Te pretre x wil now be found from the condition that dori cooling Boh cubes have the sume ccunfercntal eontrastons hee, on) =e aww sah, 2 STRENGTH OF MATERIALS from which au aE, ree In the sume manner the stest in the ste! may be ealeulated, 3 Referring to Fig. 26, what additonal tenle eres in the tube wil be produced by submiting ie to a hydrostatic inner foo Tbe: per a iif the inner diameter dh = 4 i, and b= 15/8 Xo ine? “Soluron." Cating out ofthe tube an elemental rng of wideh 1 in, the complete tensile fore in the ring wil be Bs patie sol Due to the fact thatthe unit circumferential clongations in copper and in sec are the same, the stresses will bein proportion to the Ipods ue the ste in the copper wil be s/t that inthe sec. [At the same time the cose sectional area ofthe copper is 16/8 that of the steal; hence the force P wil be equally dstsbuted between twa metals and the tensile stress im the copper produced bya Hydrostatic pressure wil be p 00 0 XRT Tw agit Bea SIT pera Foe “The sre inthe sel wil be c= ez 100 Ih. prsqin A bultap sng consists of an inner copper ring and an ooter stec rings "The inne ameter ofthe steel ng i smaller than the tute dinner of the copper ing bythe amount & and the stractare Gteembled after preirinary honing of the steel Hoge When Sole the scl ng pris pros om the copper ng ink St peste). Determine the sresses in the see and the copper fide asgembiyif both Hing have rectangular ces sections withthe dimensions by and, in sada diestion and dimensions equal Linty in the dvestion perpendiolar tthe lane ofthe rng. The “Uimensons han A may be considered small ax compared with the ‘Gameterd ofthe surface of contact of the «wo rng Sulwions Lee = be the willwmly detbuted pressure per quate inch of the surface of sontact ofthe rings; ten the cam pov tres inthe copper andthe tnale rear nthe sel wil be TENSION AND COMPRESSION 33 found from the equations: © 4 The increase of the inner diameter ofthe stel ring will be ae She, ‘The unknown presture wil be found from the equation wornnt€(cevie)s 4 from which Now the stresses e, and 2, fom eqs (a), wil be bh 2 hes ThE, {Determine the stresses which wll be proced inthe built up Fog of the previous problem by rotation of the sng with Constant speed» ‘Soluion. Duc to te fact that copper has a greater density and «smaller modulus of elasticity than steel, the copper ring wil bres onthe steel sing during rotation. Let denote the presure square inch of the surface of contact between the two igs en the corresponding stresses will be given by eqs. (@) of the Irevious problem, In addition to those Stress the steuce pro= Aced by” centage forces shosld be taken into consideration, Denocing By. and, the weights per unit volume of tel and ayes) we n(E VCE ae Combining these streses with the stress due eo pressure « and 4 STRENGTH OF MATERIALS sing ht huni cg fe things hd he sme Sent a ae a a Lee) (G*)-#]- wTu(ssty(e2 ay cat alee) (+e tom which ay bea each price Kosing Sar SSG SRE oun Mal “cameo be oe at i "Datta he ning ppl end of a oper ni thw eee ade She ccc foot “ance. = 150 feet per ss, 1 Refersing problem 2 and Fig. 26, determin the srs in the Gpper at room fomperature fi = foo" Fabremly fm he a a 20X10, Answer, 2,300 Is, per 9. in 8. Referring to problem §, determine the number of revali- tions m per minute ae which the stent in the copper ring becomes equal to aero if the inital asembly strat inthe tame Hing was 8 Compression equal toy and hm Ay and Fy 2B ‘Solution. The number of revolutions will be determined from the equation ey PAE). wen (BY [GAY (G4) Fn hsv in she ay ng of olen sig a Te REE Le ear ‘ange hs sane eat of he gs eres Really yo dre Bake aa Se nae 10" SBE poten 5 fo he ree nel and in ep eae Saas ee SP ond ip ie omen pers CHAPTER II ANALYSIS OF STRESS AND STRAIN 9. Variation of the Stress with the Orientation of the Cross Section for Simple Tension and Compression.—In ciscussing stresses in a prismatic bar submitted to an axial tension P we have previously considered (art, 2) only che stress over eross sections perpen- dicular to the axis of the bar. Wenow takeupthecase inwhich | the eross section pg (Fig. 272), perpendicular to the plane of the figure, is inclined to the axis. Sinceall longitudinal bers have the same elongation (see p. 3) theforces representing the action ofthe right portion ofthe bar on. its lefe portion are uniformly dis. tributed over the cross section pg. The left portion of the bar, isolated in Fig. 274, isin equilibrium under the action of these forces and the external force P applied at the eft end. Hence the resultant of the forces distributed over the cross section pq is equal to P. Denoting by the area of the cross section ‘normal to the axis of the bar and by yythe angle between the axis x and the normal w to the cross section py, the cross- sectional area of pg will be cos e and the stress s over this cross section is Pecos e 4 where e« = Pj denotes the stress on the cross section normal to the axis of the bar. Ie is seen that the stress ¢ over any inclined cross section of the bar ig smaller than the stress ou over the cross section normal to the axis of the bar and that it diminishes as the angle g increases. For g = x/2 the sec- econ ¢ 3) 6 STRENGTH OF MATERIALS tion gg is parallel to the axis of the bar and the stress + becomes 2e"0, which indicates that there ie no pressure be- tween the longitudinal bers of the bar ‘The stress 5, defined by equation (16), has the direction of the force P and ie not perpen dicalar to the cross se In such eases it is usual to total stress into two components, as is shown in Fig. 28. ‘The stress ‘component em perpendicular to the cross section is called the normal srs. "Tes magnitude is a = se0s ¢ = os 008 on) The tangential component + is called the shearing stress and. has the value eecosesing =Zsin2y. (18) ‘To visualize che serain which each component stress produces let us consider a thin element ext out of the bar by two ad jacent_ parallel sections pg and ‘put Fig. 292. ‘The stresses act ing on this clement are shown in Fig. 29a, Figures 298 and 29¢ are obtained by resolving. these stresses into normal and tan- sential components as explained above and show separately the action of each of these compo. rrents. It is seen that the normal direst ¢, produce extension of the clement in the direction of the normal to the erose section py and the shearing stresses produce sliding of section pg with respect to Pigie rom equation (17) it is seen that the maximum normal stress acts over cross sections normal to the axis of the bi and we have ANALYSIS OF STRESS AND STRAIN a “The maximum shearing stress, a scen from equation (18), acts cover cross section s inclined at 4§* to the axis of the bar, where sin 2¢ ~ 1, and has the magnitude (as) Although the maximum shearing stress is one-half the max- mum normal stress, this stress is sometimes the controlling factor when considering the strength of materials which are much weaker in shear than in tension, For example, in a tensile test of a har of mild steel with a polished surface, yielding of the metal is visible to the naked eye, Fig. 30. Ie occurs along the inclined planes for which the shearing stress is a maximum and at the value of the force P which corre. sponds to the point B in Fig. 2a. This indicates that in the «ase of mild ste! failures produced by the maximum shearing stress although this stress is only equal to one-half of the maximum normal stress, 38 STRENGTH OF MATERIALS Formulas (17) and (18), derived for ba in tension can bs sn he ee compres Teale tsnumed postive and compressive native, Hene for bar Under ata compression we have only to take witht negative sign in formulas (17) and (18). ‘The negative sign ae wil then indicate that Fg: Sb ve obtain instead of terion acompresive action onthe hin clement between the jee con eetione pq and por, The negative sin for + im formula (1) wl idate tat for compression ofthe bar he shearing neon on the le ee we M}) tt co that shown in Fig. 296. AUR Lt aia he ree oe AL 3 normal and shearing stress _ 1 sshich will be used. Positive sign ae for shearing is taken when they form a coupe in clockwise cei and negative ign Tor ppt areton. " 10. The Circle of Stress.—Formulas (17) and (18) can be represented graphically.! We take an orthogonal system of Coordinates withthe egin at O and weeh priv drecton of aes as a eee eros section py perpen fo the this ofthe bar ne have for this ee f= opin Fg 2) and we Bs rom fs Enmulas hind (i) en vot Selecting ale forsee ets. tring normal components slang the horton xis and sheeting comp. nents along the vere ante sos teting onthe plane with @ = ots represented in Fig. 32 by & point Phavingthesbncn equa eoerand the ordinatcequal to Sin. Takingow apane pall tothe sxisof theta wehave 2 Pe 52 "hin graphical reresetstion i do to, 0. Make, Zing sate, pe nag Sev abo he Abhandlsngan?” paige ages Hn toi ee rience ater pubesons onthe me 2bjet ae gen: ANALYSIS OF STRESS AND STRAIN » ‘¢ = 1/2, and observing that both stress components vanish for such a plane we conclude that the origin O, in Fig. 32, corre- sponds to this plane. Constructing now on Od as dlameter a circle it can be readily proved that the stress components for any cross section pq with an arbitrarily chosen angle Fig. 28, will be represented by the coordinates of a point on that circle. To obtain the point on the circle corresponding to a definite angle ¢ it is only necessary to measure in the counter-clockwise direction from the point 4 the arc sub. tending an angle equal to 2g. Let D be the point obtained in this manner; then, from the figure, OF = 00 + OF = + Fcos 2p = 12008 gy DP = TDsin 2g = % sin a. ‘Comparing these expressions for the coordinates of point D with expressions (17) and (18) ie is seen that this poine defines the stresses acting on the plane pg, Fig. 28. As the section 14 rotates in the counter-clockwise direction about an axis Perpendicular to the plane of Fig. 28, y varying from oto =/3, the point D moves from 4 to O, so that the upper half-cirele determines the stresses for all values of g within these limita. If the angle e is larger than /2 we obtain a cross section as shown in Fig. 334 cut by a plane mm the external normal, to which makes with the ¥ axis an angle larger than /3. Measuring again in the counterclockwise direction from the point 4, in Fig. 32, the arc subeending an angle equal to 20 ‘we will obtain now a point on the lower half-circle. Take, as an example, the ease when mm is perpendicular to cross section py which was previously considered. In such a case the corresponding point on the circle in Fig. 32 is point Dy such thae the angle DOD, is equal to x; thus Duis 2 diameter of the circle, Using the coordinates of point D, "The portion af the bar on which the presse act Tndicatad by ‘shading. "The external normal ye rected outward from tht parton 0 STRENGTH OF MATERIALS wwe find the stress components o., and 1 for the plane mm 65, = OF = 06 — FC = 2 — conse = ensinty (20) no FDL = - ising = -Ssinay? (a) Comparing these results with expressions (17) and (18) we find a5 ey = excost e+ ossint p= 0 22) nace (a) ‘This indicates that the sum of the normal stresses acting on two perpendicular planes remains constant and equal t0 ‘The shearing stresses acting on two perpendicular planes are numerically equal but of opposite sign. No By taking the adjacent cross sections mim. and pig: parallel to™mm an pan clement, sch ax shown in Fig 30 foolated and the directions of streses acting on this element are indiated. Tes seen that the shearing stresses acting on the sides of the element parallel to the py plane produce « couple in the clockwise direction, which, according to the accepted rule defined in Fig. 1¢, mast be considered positive. “The shearing streses acting on the other two sides of the element ana Hr, ANALYSIS OF STRESS AND STRAIN a produce a couple in the counter-clockwise direction which, According to the rule defined in Fig. 314, is negative. ‘The circle in ig. 32 called the circle of siress is used to determine the stress components em and r for a cross seetion 184 whose normal makes any angle y with the x axis, Fig. 28. ‘A similar construction can he used to solve the inverse prob Jem, when the components e, and r are given and it is re- quired to find the tensile strexe oy in the axial direction and the angle v. We observe that the angle between the chord OD and the x axis is equal tog, Fig. 32. Hence, after constructing the point D with coordinates 2, and r, we obtain fe by drawing the line OD. Knowing the angle ¢, the radius ‘DC making the angle 2¢ with the axis OC can be drawn and the center C of the circle of stress obtained. Problems 1. Determine rand r analytically and graphically ife, = 15,000 Ibs, per og ims andy = 30° of @ = 120". By using the angle o> and 120° isolate an clement at shown in Fig. 356 and show by frtows the directions of stresses acting on the element 2 Solve the previous problem assuming that instead of tensile tres there acts compresive stress ofthe sume amount. Obicr¥= that inthis case the diameter of the citee, Fig. 32, must le on he negative side ofthe abscnea, "Ona plane pp, Fig. 28, are acting a normal stress on = 12,900 Ib, per ssn. and s sheavingstree r= 4,000 Ibs per sin. Find the angle p and the ste Answer. 13,390 Ibs. per 9. 300 ie Por ey in a PH ‘Wsoibn pers}. ne '= 84s be. per agin. §. Find maximum shearing stress for the case ‘in problem 1 8: Determine the aes of cron sector whith the nama snd the soning tee re moet cally egal. 11, Tension or Compression in Two Perpendicular Di rections.—There are eases in which the material ofa structure 2 STRENGTH OF MATERIALS is submitted to the action of tension or compression in two perpendicular directions. As an example of such a stress condition let us consider stresses in the cylindrical wal of a boiler submitted to internal pressure p Ibs. per sq.in* Let us cut out a small element from the cylindrical wall of the boiler by two adjacent axial sections and by two circum- ferential sections, Fig. 342. Because of the internal pressure Yes = aw the cylinder will expand both in the circumferential and in the axial directions. The tensile stress ey in the circum- ferential direction will be determined in the same manner as in the case of a circular ring (art. 8). Denoting the inner diameter of the boiler by d and its wall thickness by A, this ow) In calculating the tensile stress ey in the axial direction we imagine the boiler eut by a plane perpendicular to the x axis. Considering the equilibrium of one portion ofthe beile i will, be appreciated thac the tensile force producing longitudinal ‘extension of the boiler is equal to the resuleant of the pressure ‘on the ends of the boiler, i., equal co ra p-o() ON “More accurately p denotes the diferenee between the intera premure andthe external atmospic pressure. ANALYSIS OF STRESS AND STRAIN “6 “The cross sectional area of the wall ofthe boiler is * A= eth, Hence : ane (2s) Itis seen tha che element ofthe wall undergoes tensile stresses a and. n two perpendicular directions The tensile stress in the Greumferental direction being twice as large athe fress on in the axial direction. We conser now the stress dover any cross section py, Fig. 4a, perpendicular to ay plane fd whose normal m makes an angle ¢ with the x axe. By sing formulas (17) and (18) of the previous article we con- clude that the tensile streses acting in the axial direction produces on the plane py normal and shearing streses of magnitude a. = oecoste, 1 = Jossin ag. (a) ‘To eaculate the stress components prodced onthe stme plane py by the tenle stress vy, we ebserve thatthe angle Beencen ey andthe normal, Fig. gaa is Z~ and is meas ured clockwise from they as, while ¢ is measured count tlockwise from the = ai.” Brom this we conclude that in ‘using equations (17) and (18) we must substitute in this case oy for and ~~ 6) instead of e This gives ea = eysinte, r= —Ioysinae. —. ‘Summing up the stress components (a) and (6) produced by and gy stresses respectively, the resultant normal and shear- ing stress components for the case of tension in the two "The thicknes of the wali teamed amall in comparnon with the iameter andthe approximate forms forthe crestor ed "'Thece in iso's prentre om the incr cyindncal aurface of the Otvervng tha this ondnte mart be taken with negative Sam, we conclue thatthe ante of the pone D, taken with the proper sen, gives the shearing sess component (2). Aon he platy tng suger ih respect to an avis perpenicalar to 4 plane, Fig 344, the Corresponding pint D is moving in the counterclockwise Giretom along te ile of ses in ig 9620 tha foreach falar of the coresponding vies of the component e+ End are obtained ae the coordinates ofthe point D. yom this graphical representation of formule (26) and (ey) llows ac once that the maximum normal stress com- oven in or gee ty and the maximum shering [os repreonted by the radius CP ofthe cre in Fg. 358 8) and oceurs when sin 2p = — 1 and y= 3e/4 The magnitude of shearing stress but with negative sign is acting fon the plane for which @ = 2/s Taking two perpendicular planes defined by the angles ‘e and #/2-+ ¢, which the normals and m, make with the = axis, the corresponding stress components are given by the coordinates of points D and D, in Fig. 3s, and we conclude eaten eet en 9) neon (9) ‘This indicates chat the sum of the normal trees ating om ‘two perpendicular planes remains constant as the angle e varies, Shearing stresses acting on two perpendicular planes are numerically equal but of opposite sign. ‘The circle of stress, similar to chat in Fig. 35, ean be con- structed also ifone or both stresses v4 and ey are compressive, itis only necessary to measure the compressive stresses on the “6 STRENGTH OF MATERIALS negative side of the abscissa axis, Assuming, for example, that the stresses acting on an element are as shown in Fig. 360, the corresponding circle is shown in Fig. g6b. The siress Re. 9 ‘components acting on a plane py with normal m are given by the coordinates of the point D in the diagram. Problems 1. The oiler shown in Fig. 3 has d= 100 iy A= 4 Determine oy and oy if p = 108 lbs. per sq. in Teoate a small clement by the planes for which g = jo and 120" snd show the ‘magnitudes and the directions ofthe seas components acting on the lateral sides af that element 2 Determine the strenet fu tay rand ry if in Fig. 368, A ste ibs. pers in. and Answer: 40 6260 bs. por st img og = ~ 150 Ths per sain r= —tm 500 I. por sin 4 Determine ony + nd th the previous problem, ifthe angle ois chosen so thats maximum, VAhoaer, ty a, = 280 Ib pet 2 iy Ibe ors aeeetie 13, Principal Stresses.—It was shown in the previous tile that for tension or compression in two perpendicular directions + and y one of the two stresses, or ob the maxi rum and the other the minimam normal stress. For all inclined planes, such as planes pg in Figs. 342 and 364, the valieof the normal tress eis Between these limiting values. =n 500 ANALYSIS OF STRESS AND STRAIN ” ‘At the same time there is acting on all inclined planes not only ‘normal stresses ¢ but also shearing stresses, r. Such stresses as ¢- and 4y, of which one is the maximum and the other the minimum normal stress, are called the principal stresses and the two perpendicular planes on which they act are ealled the principal planes. ‘There are no shearing stresses acting on these planes. In the example of che previous article, Fig. 34 the principal stresses ¢» and. were found from very simple considerations land it was required to find the expressions for the normal and shearing stress components acting on any inclined plane, such as plane pg in Fig. 342. In our further discussion (see 122) there will be cases in which it will be possible to determine the shearing and the normal stress components acting on two perpendicular planes. From the previous dis- ceussion we already know that such normal strestes do not represent the maximum stress which is the stress particularly ‘important in design. To get the maximum value of stress, the principal stresses are required. The simplest way of solving this problem is by using the circle of stress we con- sidered in Fig. 35. Assume that the stresses acting on an clementary rectangular parallelepiped are as shown in Fig. < « Nt fas ERAN, Few. 374. The stresses 2, and ey are not principal stresses, since hot only normal but also shearing stresses are acting on the Planes perpendicular to the x and y axes. To construct the 8 STRENGTH OF MATERIALS Circle of stress in this case, we use frst the stress components qe andr and construct the points D and Dy as shown in Fig. 374. Since these two points represent the stresses ecting fon two perpendicular planes, the length DD; represents a diameter of the circle of stress. ‘The intersection of this diameter with the x axis gives the center C of the circle, so that the circle can be readily constructed. The intersection points and B of the circle with the x axis define the magni- tudes of the maximum and the minimam normal stresses; which are the principal stresses and are denoted by #, and os. Using the circle, the formulas for ealeulating «, and e, can be easily obtained. From the figure we have a= 0460640 ates (4S e5, Gn she (ERY e on ‘The directions of the principal stresses can also be obtained from the figure. We know that the angle DC is the double angle between the stress ¢; and the * axis and since 2g is measured from D to. in the clockwise direction the direction ‘of 61 must be as indicated in Fig. 37a. If we isolate the ele. ment shaded in the figure with the sides normal and parallel to; there will be only normal stresses #1 and ¢s acting on its sides. For the calculation of the numerical value of the angle ¢ we have, from the figure, = 08 = 00-7 DE Regarding the sign of the angle ¢, it must be taken negative in this case since it is measured from the x axis in the clock- wise direction, Fig. 37a. Hence aye DE ang ~ = Be Ga) "The maximum shearing stress is given by the magnitude of ANALYSIS OF STRESS AND STRAIN ” the radius of the circle of stress and we have ‘The equations (31)-(34) completely solve the problem of the determination of the maximum normal and the maximum shearing stresses ifthe normal and shearing stresses acting on ‘any two perpendicular planes are given since a circle is fixed by two points at the ends of a diameter. Problems 1 An clement, Fig. 374 i submited to the action of sree 220 bs. pet 89.58 2y = 000 Tbe. pet sin 7 ~ 1000 le, sq. in. Determine the magnitudes and the directions of Principal stresses ey and os ‘Solution. By using formulas (31) and (32) we obtain onset , ae 4400 + rats = 544 Ibs. per ny 00 — Hogg = 2586 Ibs. per ag. rom formula (33) we have tnap=— aes as ye alt ‘The minus sign indicates that ¢ i measured feom the + axis inthe clockwise direction as shown in Fig. 372 1 Determine the direction of the principal streses in the previous prob Kem if'o. =~ soe Ibe persqeinn Soliton. The corresponding’ circle isshown in Fig. 8, tan bp = ele |, dem 142". Hence the ange which he 74 I ‘aximum compressive stress makes with thease equal to 7" and is measured re ‘couneerclockwse from the a 3. Find the ‘incl of stress for the Fee case of two equal tensions ¢. = ¢y 57 And for two equal compressions soy =~ ¢. r= ain both cases, nse Circles become points om the horizontal axis with the abscissas o and ~ o respectively so STRENGTH OF MATERIALS 4 On the sides of the element shown in Fig. 3g are acting the streiies 0. — 500 Ibs pet oq. ty ay = 1600 Tbe. er ing += 1,000 lbs, per aq. in Find, by uslng the circle of tres, the Imagnitudes ofthe normal and shearing streses on (e) the principal planes, (0) the planes of maximum shearing etree chin, Ee creping ice cf res shown i Fig a ‘The points D and D, represent tresses acting on the sides Of the clement in Fig. 39a perpendicular to the vand.y axes OB and O4 fepresent the principal streses. Their magnitades are ey = 1yoty Ibs per aq, ine and on = ~ 914 tbs per sq. in. respectively. ‘The Airgtion of the maximum compressive ses o; makes the angle = 121" withthe raul this angle being measured Irom the + sx in the counter-ciockwive direction as shown in Fig. 39a. The points Pe 9 and Fy represen streses acting on the planes subject to maxim ‘hear. The mageitade of this shear tpt Ibs. por seine OC ‘represents the nial steste gual to 500 fb pete in acting on the tame plane. Save the previous problem if 2g = — 5,00 bs. per ny 500 Ih per. ny = 100 ipo i. 14, Analysis of Strain in the Case of Simple Tension — In article 2, the axial clongation of a bar in tension was discussed. Experiments show that such axial clongation is always accompanied by lateral contraction of the bar, and ‘nit lateral contraction , : that the ratio SUSE HREEECTO i constant for a given bar within the elastic limit. This constant will be called ‘8 Poisson's rato, after the name of the French in who determined this ratio analytically by ANALYSIS OF STRESS AND STRAIN st vsing the molecular theory of structure of the material. For materials which have the same elastic properties in all diree- tions, so-called isotropic materials, Poisson found x = 1/4. Experimental investigation of the lateral contraction in struc- tural metals’ shows thae ais usually not very far off che value calculated by Poisson. For instance in the case of structural steel it can be taken asx = o.30. Knowing the Poisson ratio fof a material, the change in volume of a bar in tension can be calculated. The lengeh of the bar will increase in the ‘The lateral dimensions diminish in. the 2:1. Then the volume of the bar io (1-+4)(1— no: 1, which becomes (Ce 24) £1 iF we recall that eis a small quantity and neglect its powers. Then the unit volume expansion is «(i — 29). Teis unlikely that any material diminishes its vol- lume when in tension; hence a must beless than o.so. Forsuch + materials as rubber and parafin » approaches the above limit and the volume of these materials during extension remains approximately constant. On the other hand such material as concrete has a small magnitude of w (u = 1/8 t0 1/12) and for cork y can be taken equal to zero. The above discussion of lateral contraction during tension ‘ean be applied with suitable changes to the cave of com- pression. Longitudinal compression will be accompanied by lateral expansion and for calculating this expansion the same value for was in the case of extension is used. Problems 1, Determine the increase in unit volume ofthe bar in tension if 9. = 600 Ibs. pers. wm 0:39, Bm 30-40! Ie. per 9. im dies ET af a foo “nw -¥ AS 00) my x10 2. Determine the increase in volume of a bar the extension of "These materials canbe considered asta (oe Part I, 2 STRENGTH OF MATERIALS which is produced by the force P atthe end and the weight of the Ter ge le) ae : Answer. "The incense in volume is equal to #02 4) 15, Strain in the Case of Tension or Compression in Two Perpendicular Directions.—If a bar in the form of a rec tangular parallelepiped is submitted to tensile forces acting in two perpendicular directions « and y (Fig. 4), the elongs- tion in one of these directions will depend not only upon the tensile stress in this direction but also upon the stress in the perpendicular direction. The unit elongation in the direc- tion of the s axis due to the tensile stress ¢- will be ox/E. The tensile stress oy will produce lateral contraction into # direction equal to pes[E; then if both stresses #, and oy act simultaneously the unie elongation in x direction will be <0 Gs) un} Similarly, forthe y direction, we obtain “E- 66 oF In the particular ease, forthe ewo tensions equal, = we obtain, w= Ew Gn From eqs. (35) and (36) the stresses ex and ey can be obtained ts functions of unit strains e, and «, as follows: (a+ mE (ot nae, 3 co) If in the eaze shown in Fig. 392 the elongation «in axial iretion ad the elongation « in creumferential direction are measured by an extensometer the corresponding tendle sreses sand sy will be found from equations (38) ANALYSIS OF STRESS AND STRAIN 58 Problems 1, Determine the increase inthe volume of the cylindrical steel boiler under internal pressure (Fig. 34), neglecting the deforms (ofthe ends and eaking wy = 6209 be. pers in. Solution. By using eqs. (93) and (98) 002 5 5 BOO 5100 7 Bexid 95 Kid” jee = TX ITH = te _ 4, S00 100. Spx SPR” PRT The volume ofthe boiler will increase inthe ratio (Rad baler = (bet ag) st HooegB 2A cube of concrete is compressed in two. perpendicul directions by the arrangement shown in Fig. 42. Determine th ‘decrease in the volume of the tube if itis 4 inches on & side, fhe compressive stress is uni= > formly distributed over the faces, peo and P = 20,00 Ibe. Seluion. Neglecting Fiction in the hinges and considering the - ‘equilibrium of each hinge (Fig. a = sepa Cee hag focoeionae ae Seahorse so" ~exyxil Seep pepo pe oe Thera pr et lan of he oe abbr samen bee ts x 0.1) = = oonngyt. = oco00t, st STRENGTH OF MATERIALS Solution, Increase per unit area of lateral surface = 4. + 6 rx to “s Determine the unit elongation in theo. direction of bar of seal ifthe stress conditions are suchas indicated in problem 1,49 ‘Solution « (alg ~ 03 X 2586) = 1546 x 10-4 “px 16. Pure Shear. Modulus in Shear.—Let us consider the particular ease of normal stresses acting in two perpendicular directions in which the tensile stress 6 in the horizontal di- rection is numerically equal to the compressive stress ¢y in the vertical direction, Fig. 41a. ‘The corresponding circle of ! SS “a T stress is shown in Fig. 414. Point Don this circle represents the stresses acting on the planes ab and ed perpendicular to xy plane and inclined at 45° to the * axis, Point D, represents stresses acting on the planes ad and Je perpendicular to ab and cd. eis seen from the circle of stress that the normal stress on each of these planes is zero and that the shearing stress over the same planes, represented by the radius of the a > on Combining the effects of the forces P,, concluded that over a section through the 7 axis only the forces P, and P, produce stresses and therefore these stresses e STRENGTH OF MATERIALS may be calculated from eqs. (26) and (27) and represented ‘graphically by using the Mohr circle. In Fig. s1 the stress circle with a diameter 4B rep- resents these stresses, In the foe same manner the stresses over _[, iY sion though the = as ]- can be represented by a circle having BC as a diameter. ‘The circle with the diameter AC represents stresses over any sec- + ——* tion through the y axis, The Pre se three Mohr circles represent stresses over three series of sections through the x, y and = axes, For any seetion inclined 0 x, y and 2 axes the stress components are the coordinates of point located in the shaded area of Fig. 51." On the basis of this it ean be con- cluded that the maximum shearing stress will be represented by the radius of the largest of the three circles and will be given by the equation tau = (#2 — Te will ect on the section through the y axis bisecting the angle beeween the + and 2 axes. ‘The equations for calculating the unit elongations in the directions of the x, y and 2 axes may be obtained by combin- ing the eects of P., Py and P, in the same manner asin con- sidering tension or compression in two perpendicular directions (cee article 15). In ehis manner we obtain en Gobet (a) en Gf te. Whe prof ofthis rtement canbe found inthe book by A, Foppl, ‘Technische Mechanie, Ve. 7. 18,1016. See also H. I, Westerpuard, ange. Math. Meck, Val’, p39, 924 ANALYSIS OF STRESS AND STRAIN 63 ‘The volume of the bar increases in the ratio GFOUtOUtA EE oF, neglecting smali quantities of higher order, GO teta torn Te is seen thae the unit volume expansion is etete Gs) ‘The relation between the unit volume expansion and the stresses acting on the sides of the bar will be obtained by adding together eqs, (43). In this manner we obtain drotete CSD eta tel 4s) In the particular case of uniform hydrostatic pressure we have onan na np ‘Then from eas. (43) an yaa — 20 -, GO). and from eqs. (44) ane we or, sing the notation x, «®) weobtain a=-§% (9) ‘The unit compression is proportional tothe compressive stress ‘p and inversely proportional to the quantity K, which is called the modulus of elasticity of lume. o STRENGTH OF MATERIALS Probleme 1, Determine the decrease i the volume of slid stelephere of 10 inch diameter submitted 0 uniform hydrostatic presse (P= yoo Ibe. per 2g. inch ‘Slaton. Frome, (49) ge Bn Teme X a2 X09) x RIF ia “The decreate in the volume is, cherefore, re Ex = aroy cubic inch, 2. Refering to Fig. $24 rubber cylinder is compressed in a steel epinder B bya are P. “Determine the presse between the ‘uber ad the stl iP = 1050 Ib "3 ney Passon's ratio for rubber p 045.” ‘ction be. tween rubber and wel neglected Soltion. Lat denote the compressive stressen cover any eras section perpendicular to the is of the eylinder and the presse Between the rer land the inner susface of the stel cylinder Com: Dresive stress of she sane magnitade wl act be. Eroen the lceralsrfacer ofthe longitudinal bers oF the rubber eylndr, fom which we flats an xy clement nthe form of etangalaeparallepipe, sich tides parallel co che axis of the eyindor (ee Fig. sa. his lemene iy equilibrium under np. the compretve stress gon the lateral faces he eneneand th nial conpresivestes. Aasum- dng thatthe tel cylinders absolutely igh, the lateral expansion ‘he ruber inthe andy direction shouldbe equal © seo and From eas (4) we ebtain o=£-So49. 4. Aconeretecolumnisenclosedina steel tube (Fig. §3). Deter imine the presure between the steel and concrete and the ccurnfer ANALYSIS OF STRESS AND STRAIN 6s pee oe Ser See ae oe en ie Rogen ao an - E+E Gt. oO ms ‘This expansion should be equal tothe circumferential expansion of the tabe (ee eq. 13) o From eqs. (a) and (2) we obtain Lee area UE Eee Hh from which ace ae getinm he crcumfeenia eile tres inthe cube will ow be cele fom equation at, and, 4 Determine the maximom sheating stress in. the concrete column of the previous problem, assuming that p = 1000 Tos, perag. in, nem Ba, dak 75. Selatan vm (8 = 74 Ibs. per qin. CHAPTER IIT [SHEARING FORCE AND BENDING MOMENT 19. Types of Beams.—In this chapter we will discuss the simplest types of beams such as shown in Fig. 54. Figure 542 : represents a beam with simply [P yy supported ends. Points of sup- le port 4 and B are hinged 30 B, that the ends of the beam can —"" rotate freely during bending. eis also assumed that one of the supports is mounted on rollers and can move freely in the horizontal direction. Fig. ture 548 represents a camtileser _ Beam.” The end dt of this beam a is buile into the wall and eannot rotate during bending, while the end B is entirely free Figure s4¢ represents a beam with an overhanging end. ‘This beam is hinged to an immovable support at the end and rests on a movable support at C. All three of the foregoing cases represent statically de- serminate beams since the reactions at the supports produced by a given load can be determined from the equations of statics. For instance, considering the simply supported beam carrying a vertical load P, Fig. 59, we see that the reaction Reat the end B must be vertical, since this end is fre to move horizontally. Then from the equation of staties, 2X’ = o, it follows that reaction Ris also vertical. The magnitudes of Ry and Ry are then determined from the equations of mo- ments. Equating to ero the sum of the moments ofall forces with respect to point B, we obtain Ry Pb=0 SHEARING FORCE AND BENDING MOMENT 67 from which Ina similar way, by considering the moments with respect to point 4, we obtain ‘The resetions for the beam with an overhanging end, Fig. $4¢, ‘can be calculated in the same manner. In the case of the cantilever beam, Pig. 54, the load P is balanced by the reactive forces acting on the builtin end. From the equations of statics, EX = 0 and ZY = 0, we con. clude at once that the resultant of the reactive forces Ry must be vertical and equal to P. From the equation of moments, 2M = o, it follows that the moment M, of the reactive forees with respect to point 4 is equal to Pa and acts in the counter- clockwise direction as shown in the figure. ound in, bridges. “In beams the above types of beams can be calculated by a similar free rotation of the ends and free motion ofthe support are of shorter span, the conditions bending of sucha Beam friction : Produced such as to oppose rotation and horizontal move- sce. 178; but for rigid beam the defection of which is very the beam were simply supported, Fis. saa @ STRENGTH OF MATERIALS 20. Bending Moment and Shearing Force—Let us now considera simply supported beam on which act vertical forces Pi, Ps, and Pr, Fig. 56a. We assume that the beam has an faxal plane of symmetry and that the loads act in this plane ‘Then, from considerations of symmetry, we conclude that the bhending also occurs in this same plane. In most practical cases this condition of symmetry is fulfilled since the usual crose-sectional shapes, such as a circle, a rectangle, an I, oF aT, are symmetrical. The more general case of a non- symmetrical cross section will be discussed later (see p. 93). 7 on jesus 5 ‘ Ee = It ——.— —_—" ee To investigate the stresses produced in a beam during bending, we proceed in the same manner as we have already used in diseussing the streses produced in a bar by a central tension, Fig. 1. We imagine that the beam 4B is eut in two pparts by a cross section mm taken at any distance s from the lefe support 4, Fig. 6a, and thatthe right portion ofthe beam is removed. In discussing the equilibrium of the remaining leftchand portion of the beam, Fig. s6b, we must consider not ‘only the external forces such as loads Py, Ps, and reaction Ry bbut also the internal forces which are distributed over the cross section mmr and which represent the action of the right portion of the beam on the left portion. These internal forees SHEARING FORCE AND BENDING MOMENT 6 must be of such a magnitude as to equilibrate the above men- toned external forces Py, Ps, and Ry Tn che enauing dicossion i wll be advantageous to reduce the actual system of external forces toa simplified equivalent system. From statis we know that a system of parallel forees can be replaced by one force equal tothe algebraic sum ofthe given forces togcther with a couple. In our particular ease we ean replace the forces Py Ps, and Ry by the vertical force 7 ating in the plane ofthe cross section mn and by the couple M. ‘The magnitude ofthe force is V=R-P-Py o and the magnitude of the couple is M ) = Pale = a) o oe = Pe = ‘The force V7, which is equal to the algebraic sum of the ex- ternal frees to the left of the cross section mn, is called the shearing force at the cross section mm. The couple M, which {sequal to the algebraic sum of the moments of the external forces to the left of the cross section mm with respect to the centroid of this cross section, is ealled the Bending moment at the cross section mm, ‘Thus'the system of external forees to the left ofthe ross seetion mi can be replaced by the statically equivalent system consisting of the shearing force 7” acting in the plane of the cross section and the couple M, Fig. 6c. ‘The stresses which are distributed over the cross section mr and which represent the action of the right portion of the bbeam on its left portion must then be such as to balance the bending moment M and the shearing force V” Ifa distributed load rather than a number of concentrated forces acts on a beam, the same reasoning can be used as in the previous case. Take, as an example, the uniformly loaded beam shown in Fig. s7a.. Denoting the load per unit length by g, the reactions in this ease are 70 ‘STRENGTH OF MATERIALS ‘To investigate stresses distributed over a cross section mn we again consider the equilibrium of the left portion of the beam, Fig. 578. Theexternal forces __}& acting on this portion of the TIMI, _s beam are the reaction Ry and the load uniformly distrib- uted along the length x. This latter load has, of course, 2 resultant equal to gs. The algebraical sum ofall forces a to theleft ofthe cross section mn is thos Ry — qe. The algebraic sum of the moments ofall forces to the left of the ‘ross section mr with respect to the centroid ofthis cross sec- tion is obtained by subtracting the moment of the resultant of the distributed load from the moment Rex of the reaction, ‘The moment of the distributed load is evidently equal to o ext Thus we obtain for the algebraic sum of the moments the Pee ante All the forces acting on the left portion of the beam can now be replaced by one force acting in the plane of the cross section ‘mn and equal to rR w~a($-x) © together with a couple equal to M=Rx-Z= 80-4 @ ‘The expressions (¢) and (4) represent, respectively, the shear- ing force and the bending moment at the cross section m. In the above examples the equilibrium of the left portion of the beam has been discussed. If, instead ofthe left portion, SHEARING FORCE AND BENDING MOMENT 71 the right be considered, the algebraic sum of the forces to the right ofa cross section and the algebraic sum of the moments of those forces have the same magnitudes Vand M as have already been found but are of opposite sense. This follows from the fact that the loads acting on a beam together with the reactions Ry and Re represent a system of forees in equi librium and the moment ofall these forces with respect to any point in their plane, as well as their algebraie sum, must bbe equal to zero, Hence the moment of the forees acting on the left portion of the beam with respect to the centroid of ‘ross section mu must be equal and opposite to the moment with respect to the same point of the forces acting on the right portion of the beam. Also the algebraie sum of forces acting on the left portion of the beam must be equal and opposite to the algebraic sum of forces acting on the right portion, In the following discussion the bending moment and the shearing force ata cross section ma are taken as positive if in considering the left portion of a beam the directions obeained are such as shown in Fig. ¢7c. To visualize this rule of sign for bending moments, let us igolate an element of the beam by two adjacent cross sections mn and mins, Fig. 58 If che cf D Pes bending moments in these cross sections are positive the forces to the left of the cross section mn give a moment in the clockwise direction and the forces to the right of the cross section mim a moment in the counter-clockwise direction as shown in Fig. s8a. Tes thus seen that the directions of the moments are such that a bending is produced which is convex downwards. If the bending moments in the cross sections n STRENGTH OF MATERIALS smn and mum are negative, a bending convex upwards is produced as shown in Fig. §8. ‘Thus in portions of a beam where the bending moment is postive, the deflection curve is convex downwards, while in portions where bending moment is negative the deflection curve i convex upwards c ‘The rue of signs for shearing i forces is visualized in Fig. 59. a 21. Relation Between Bend- ing Moment and Shearing Force. —Let us consider an element of beam cutout by ewo adjacenteross sections mm and myn which are a distance dv apart, Fig. 60. Assuming that there isa positive bending moment and a positive shearing force at the cross sec- sion mo, che action of the lft portion ofthe beam on the clement is represented by the force V and the couple M as indicated in Fig. 602. In the same manner, assuming that at section f5 o man the bending mament and the therng force ar postive, the action of the right portion of the beam on the element is represented by the couple and the force shown. If no forces Acton the beam between cross sections mu and mm, Fig. 60a, the shearing forces at these two cross sections are equal! ~The weight ofthe lant of the beam ir neglected in tie cwsion SHEARING FORCE AND BENDING MOMENT 73 ‘Regarding the bending moments, it can be seen from the equi- librium of the element that they are not equal at two adjacent ‘ross sections and that the increase dMin the bending moment ‘equals the moment of the couple represented by the two equal and opposite forces 7 ie, aM = Ve fa de and v 50) ‘Thus, on all portions of a beam between loads the shearing force’is the rate of change of the bending moment with respect t0 Let us now consider the case in which » distributed load of intensity q acts between the cross sections mn and man, Fig. 606. Then the total load acting on the clement is ade. fg is considered positive when the load cts downward, ¢ may be concluded from the equilibrium of the element that the shearing. force at che cross section myn is different from that at m by an amount a =~ 4s, from which ie follows that wv Gane ro) ‘Thus the rate of change of the shearing force is equal to the intensity of the load with negative sign. ‘Taking the moment of all forces aeting on the element wwe obtain nt = Poe ~ gtx Neglecting the second term on the right side as a small ‘quantity ofthe second order, we again arrive at equation (30) and conclude that the rate of change of the bending moment is equal to the shearing force in the case of a distributed load as well " STRENGTH OF MATERIALS If a concentrated load P acts between the adjacent cross sections mm and min, Fig. 60, there will be an abrupt change in the magnitude of the shearing force. Let denote the shearing force at the cross section mn and V’, that atthe eross section mm. Then from the equilibrium of the element mann, we find Wav HP. ‘Thus the magnitude of the shearing force changes by the amount P as we pass the point of application of the load. ‘ean then be concluded that at the point ‘of application of a concentrated force theres an abrupt change in the magnitude of the derivative dif/ds. 22, Bending Moment and Shearing Force Diagrams. Je was shown in the preceding discussion that the stresses acting on a cross section mm of a beam are such as to balance the bending moment M and shearing force Vat that cross section. Thus the magnitudes of M and Vat any cross section entirely define the magnitudes of stresses acting on that cross section. To simplify the investigation of stresses in s beam it is advisable to use a graphical representation of the variation of the bending moment and the shearing force along the axis of the beam. In such a representation che abacissas indicate the position of the cross section and the ‘ordinates, the values respectively of the bending moment and shearing force which act at this cross section, positive values being plotted above the horizontal axis and negative values below. Such graphical representations are called bending ‘moment and shearing force diagrams, respectively. Let us consider, as an example, a simply supported beam with a single concentrated load P, Fig. 61° The reactions in this cage are Pb Pa Ra and RA ‘Taking a cross section mm to the left of P, it can be concluded be omitted SHEARING FORCE AND BENDING MOMENT 75 that at such a cross section vaPt and The shearing force and the bending moment hae the ame tre teen Figures sh and ote hee ee Ber es wen tne he entog ore ae ee Sing prton ofthe team othe leo et se ae the fending moment vats dey as x. for te te monet stand rs fest cheers een Shee nd apes eh moment equals Publ The oes toning porns ofthe shring face ad ening ase Eegrant ore chown in Fg Bd tad ne encom ee @ eee 4a flee eee (nme o ny @ o straight lines ae and aye. For a cross section to the right of the load we obtain PB vate Pa ~F and Mae Pea), x always being the distance from the left end of the beam, ‘The shearing force for this portion of the beam remains 16 STRENGTH OF MATERIALS constant and negative. In Fig. 612 this force is represented by the line ’ parallel to the. axis. ‘The bending moment is a linear function of x which for x ~ a is equal to Padil and for x = Tis equal to zero. Its always positive and its varia. tion along the right portion of ehe beam is represented by the straight line 64s. The broken lines acc’ and ayes in Figs. 1b and 61e represent respectively the shearing force and bending moment diagrams for the whole length of the beam. Ae the load P there is an abrupt change in the magnitude of the shearing force from the positive value Péj/ to the negative value ~ Paj! and a sharp change in the slope of the bending moment diagram. Tn deriving expressions (8) for the shearing force and bending moment, we considered the lef portion of the beam, 1 portion which is acted upon by the two forces Ry and P. Te would have been simpler in this ease to eonsider the right portion of the beam where only the reaction Pail acts. Fol lowing this procedure and using the rule of signs indicated in Figures 38 and 9, we obtain pate aurea) Expreions (6) previously obtained canal be brought to this simpler form if we observe thae a ie nintaertng ts soe thatthe seaing force dagram consists of two rectangles the areas of which are equal. ‘Taking into consideration the opposite signs ofthese areas we conclude that the total area of the shearing force diagram is zero, This result isnot accidental. By integrating equation (so), we have flare fire, o where the limits and B indicate chat the integration is taken over the entire length of the beam from the end 4 to theend 2. The right side of equation (d) then represents the total area of the shearing force diagram. The lefe side of SHEARING FORCE AND BENDING MOMENT 77 the same equation, after integration, gives the difference My — Mz of the bending moments at the ends B and In the case of a simply supported beam the moments at the ‘ends vanish; hence the total area ofthe shearing force diagram vanishes. If several loads act on a beam, Fig. 62, the beam is divided into several portions and exprestions for V and M must be * “4 Pe 6 established for each portion. Measuring x from the left end of the beam and taking « < as, we obtain for the ist portion of the beam Roo and M= Re. o For the second portion of the beam, ie for ay <# 8 femile stress is one third of the maximum compresive stress. ‘The depth ofthe beam & = 4in., the thickness of the web and ofthe flange? = # ine Solution. “in onder to satisfy the conditions tis necessary for the beam to have dimensions such tha the distance ofthe censroid STRESSES IN LOADED BEAMS ot fom the eae bxom ge wil ath the euton ¢= forsee ee ine he eld dak ron dees, wheenof Ee from which sin ay fat 42. Determine the ratio (e)nm (xan for & channel, Bent 38 shown in Fig 85,7 = 2, = toingd = 24 in, “Maroon Ty nas! (Jun = 33 = 7: q 3, Determine the condition at which the di a -minthing of the depth fof the section shown in Fig. 861s accompanied by an increae in section nodule. ree Solution Fra 86 bee “The condition fr inereas in with decrease of ht ae dh oe er 4. Determine what amoune should be cut from an equilt- eral wiangular cross seein (ig. 83,0) in oder to tan the mat ‘mom 2. 5. Determine the rato ofthe might of thee beams of the sane length under the same 3 and (ene and having a eu Sections respectively a ctl, aequare anda rectagle with pro Portions 2 3 “elution nant req93. 2 STRENGTH OF MATERIALS 25, General Case of Traneverslly Loaded Beams. In the general eae of transeraly fed beams the seace Aixenbuted over « rom secon of« eam mun bance the Shearing force snd the bending momenta that eros sao ‘The caleaaton ofthe ereesouualy made'n ovo sets by determining fst the stesesprodaceé by the Bending mo. ten, the called dendng sess, and aterwants thoes, ing scree proce by the shearing force, Tn this ae we Shall limie ourself othe calealacom of the Bending sess the disunson of shearing strsey will be gven i the nex article. Tn caleulatng bending sees We assume hat these steses are dstfbuted inthe same manner au the eee of pare bending snd wil se she formulas derived for he streses Inert 3. Experiments show that sucha proceore giver satsfactory rete f we ae deaing with sechons wich oe tot very clove tothe point of aplication af coneeneated forex Inthe vicinity othe applcatonof a conentatel nad the stes dwbtion is more completed. This probes wll be diced in Part The calculation of ending stretes is usualy made for the cross seedons at which the bending moment hes the Ingest posive or nyative values Having the momar maim of te bending moment andthe magtatd of the Alowable sre im ening, the required eros section dimensions ofs eam can be obtained fom the equation a 6) The application of this equation will now be shown by a number of examples. Problems 1 Determine the nctay dinesin of standard T bam to support s distributed load of 4o0 Ibm. per fot, ss showa in Fg wien the working stress 2 = 16,000 Tbe, per say im, Only the normal stress e, are to be taken into consideration and the weight ofthe bean is ogleted STRESSES IN LOADED BEAMS 105 Solution. To olain the dangerous seton she sheting foe diagram should be constructed (Fig. 37,0). "The reaction at the t lefe support is BX 4X 4X Ry = BAD H ISA 6 roy, “The shearing force for any crocs cection of the portion AC of the beam is = Ry ge = 3970 — 400 X x ‘This force is zero for x = 3,770/400 = 943 feet. For this section the bending moment isa maximum, M ox 94 — 0X1 oat SRE CT] = “The necessary setion modulus 09 X12 1600 This contin insted by «standard Ee z = 134int Tham of depth 8 ns croectonal area ‘a, iny aod Z —'nyains 4 SSA oaden dam (hip 1) consists of Feat wera bare such as a of reeangular {rons ection and dimeson h = 1 fot supported a the ends, De- fering (cron the length of the bars = ect and the weight ‘of the bare neglected Solution." isthe width of one bar, the complete hydrostatic pressure on the bar, represented by the triangular prism BG, is 108 STRENGTH OF MATERIALS = 180% Gags. The reaction at Ais Ry = YF = WP x 624 Ibs. and the shearing force at any cros section wm io equal to the reaction Rt minus the weight of the pris. mn of ater, hes veR-¥ ‘The positon of the cross section coresponding to Mua is found from the condition I” = oof foom which ‘The bending moment at any cross ection mis equal to the moment ofthe reaction R minus the moment of the distributed lod repre= ‘sented by the eangular prism mn. ‘Then = -£). to. feet, we obtain ‘Substeuting, fom the above 1 = § and Mane = HP X 6204 X 10.4 bs. feet, Siow 2(1 fa D108, Sti = 2 (5 PARI ote pra. 43. Determine the magnitude ‘of Muu in a beam loaded by a triangular load ADB equal to 1 13400 Iba if = 12 feet and 3 foce (Fig. 8). Solution. “The distance © to the vertical through the center of travity C from the support Bis, in the cate of ta ent o as The reaction a the support ie then _ ex BS 500 Ibe, STRESSES IN LOADED BEAMS ws ‘The shearing force at any cross section mn sequal to the reaction Rt minus the weight ofthe load represented by area fmm Since the load represented by the area waa ADE ~ in, we obtain es Hews from which = 67 fet. ‘The bending moment atany crows section minis equal to the moment of the eaction Ry minus the moment of the load mm. “Then M Sobsticting forthe above value Maus = 22,490 Ibs sc Conarur the ending [I fp « moment and shearing force dit ee beg te trams for the case in Fig 90 (6) = nd determine the necessary Standard T beam if "= fh. = Gicet, P= 000 bsg = 420 a Ibs. per oot, eu'= 15c001bs.per J seine The weight ofthe betn ‘an be neglected i ‘Solution. In Fig. go (8) and INTE (2 “the bending. moment and shearing force diagrams pro- ¥ duced by the distributed loads ras, Ste shown. To this the moment ‘the shearing ferce produced by P should be added. The 106, STRENGTH OF MATERIALS ‘maximum bending moment willbe at the middle ofthe span, Ma = 19,200 Ibs ‘The necessary 2a EXE ie igs 159900 ‘The standard I beam of depth 8 in. and crosesetior sq. ny Z= 16.1 int is the nearest cross section sa strength conditions. Determine the most unfavorable postion of the hoisting carrage ofa crane which rides om a bears ae Fig. gt. Find Mos, iF the load per whed! ie P'= 10,000 Ibo 1m 34 foot, d= 6 feet, The weight Pf the beam is neglected. Soliion. If ie the distance of the lefe wheel fiom dhe lefe support of the beam, the bending moment wader this we Me, 7 ‘This moment bevomes 2 maximum when La hence in order to obtain the maximum bending moment under the left wheal the carsage must be diplaced from the middle position Dy a distance dja cowards the right support. The same magnitude of bending moment can be obtained also under the righe wheel by Miplacing the carriage by da fom the middle position towards the lef suppor reer ee t te Po. 6 Thersisof crane (Fig. ga) re supported by two standard I beams Determine the most ufvorable postion ef the came the STRESSES IN LOADED BEAMS 107 ‘corresponding May and the dimensions ofthe I beams ifew = 15,000 lbs. pe sq! ="3ofeet,a = ta fer, d = fleet, the weight othe erane 1¥'= 16,200 lb, the load ited by the erane P= Soc Ih ‘The loads are acting in the middle plane between the two t beams and are equally distributed between them, Solution. "The maximum bending moment will be under the fight wheel when the distance ofthis whel fom the right supports equal t0 f= 10 bd); Mom = 13009200 Ibe. in. Dividiog the ‘moment equally between the two beam, we find the necessary Man 2 Meo s36in2 ‘The necessary I beam has a depth 2 in, cross-sectional area 9.6 ing 2= 3h int The weight ofthe beam neglected, 7A circular wooden beam supported at Cand stacked to the fonction at 4 (Hg. 95) caves load g = oo lbs pet fot se : ar i a formly distributed along the portion BC. Construct the bending ‘moment diagram and determine the necessary diameter af ae ace Ibs. per oq ina a = 9 fet, b= eee Solution. “The bending inomene diagram ie shown in Fig, 93 ), "Numerically the largest moment will be at © aod is sual {0 6480 Ibe. int 108 STRENGTH OF MATERIALS 4, The wooden dam backed by verti pillars builtin at the lower ends (Fig. 9a) consists of horizontal bounds Determine the dimension of the square cross section of the pilars if = 6 feee, feet and sy = $20 Tb. per ogi. Construct the beading rmomient and sharing fore diagrams: “Solution. "The total lateral load on one pile i epresented by the segh ofthe eager da wate, Atay rs section nt, the shearing force and the bending moment are wot we x AATBH ER EAdts Tn determining the signs of 7 and. M itis asumed thas Fig 94 6 rotated go" in the counter close direction so as to brig the faca# andy into coincidence with those of Fig. $6. "The necsary dimension Bis found from equation 1KOX 64X12, = v 990i ‘The construction of diagrams is left to the reader. ‘> Determine the necessary dimensions of = cantilever beam ‘of a/standard T section which carries « uniform load y = 200 lbs. per foot and a concentrated load P= soo Ibe at the end if the IEngth = oct and ew = 15,009 eto salon (522 § + 1900 X 29)13 y= SOX $4 100 KANT ing 15,500 “The necessary standard I beam is § in. deep and of 28 sectional are. Ser to. Determine the bending stresses in a rvet by assuming that the loads acting on the rivet are distributed ae shown in Fig. 95 STRESSES IN LOADED BEAMS 109 ‘The diameter of the river = alg ing A= P= tooo lb. per aq. is Solution. “The bending moment at the cross section mn ie Pia X Ala.” The bending moment a the middle cross section mn, G24) the stresses. Then ad (edna B($4 8) 282 BEM ota prin me Ds , ai fa simp ; beam of standard section sach stto cary 4 uniform loud of 403 seers 4 but also shearing stresses + f are produced in any cross sec tion mor of the beam, Fig. 96 2 Considering the setion om the cL. right portion ofthe beam, Fig. 96 i ean be concluded from = the conditions of equilibrium is thatthe magnitude ofthese shearing stress is uch that their summation is equal tthe shearing force In investigating the law oftheir distribution over the area of the eros section vee begin with the simple ease of rectangular crows section, Fig. 97. In such case it is natural to assume thatthe shear la in An = 3/8 ing, no STRENGTH OF MATERIALS ing stress at each point of the cross section is parallel to the shearing force V, i.e, parallel to the sides mn of the cross section. We denote the stress in such a case by tu The subscript y in tye indicates that the shearing stress is parallel to the y axis and the subseripe x thatthe stress acts in a plane perpendicular tothe» axis. Ava second assumption we take Fra 9. the distribution of the shearing stresses to be uniform across the width of the beam ce... These two assumptions. will enable us to determine completely the distribution of the shearing stresses. A more elaborate investigation of the prob. Jem shows that'the approximate solution thus obtained is usually sufciently accurate and that for a narrow rectangle (large in comparison with 4, Fig. 97) it practically coincides with the exact solution? If an element be cut from ehe beam by adjacent cross sections and by adjacent planes parallel to the neutral plane, "The oct saluton of his problem is doe to de Saint Vennt, Fowrnal ‘de Mth (Liouville, 1866. Am account of Se Stine Vsunts teers rrp Ton nd Haier fh ey ef Flasicey-” The sppronmate elucon piven blow by fours. For he French ransatio of work se Ale dor Ponta Chacsces, 4856. The exact theory shows tht when the dnt ofthe beam sal in comparison withthe wath the dacepency between the esac sd the sbpronmete theres boomes considerable STRESSES IN LOADED BEAMS um as in Fig. 97 @), in accordance with our assumption there is a uniform distribution of the shearing stresses ts over the vertical face acciss. ‘These stresses have a moment (raddy)de about the lower rear edge ee of the element which rust be balanced by the moment (rabds)dy duc to shearing stresses distributed over the horizontal face of the element, ediéey. ‘Then rubdyde = rabsdy and tye = toy » the shearing stresses acting on the two perpendicular faces of the element are equal. The same conclusion was met before in simple tension (see p. 40) and also in tension ‘oF compression in two perpendicular directions (see p. 49). ‘The existence of these shearing stresses in the planes parallel to the neutral plane can be demonstrated by simple experi- ments. Take two equal rectangular bars put together on simple supports ss shown in Fig, 98 and bent by» concentrated load P. If there is no friction be- Lf tween the bars, the Bending of in each bar will be independent of “> oe that of the other; each will have : compression of the upper and ten- sion ofthe lower longitudinal bers \t and the condition will be that in- or dicated in Fig. 98 (2). The lower Pe Tongitudinal fibers of the upper bar slide with respect to the upper fibers of the lower bar. In a solid bar of depth 2% (ig. 98, «) there will be shearing stresses along the neutral plane 2m of such magnitude as to prevent this sliding of the ‘upper portion of the bar with respect to the lower, shown in Fig. 98 (@). Due to this prevention of sliding the single bar of depth 24 is much stiffer and stronger than two bars each of depth A. In practice keys such a8 a 8,6. + are sometimes used with built-up wooden beams in order to pre- vent sliding (Fig. 99, 2). Observation of the clearances around a key, Fig. 99 (8), enables us to determine the diree- tion of sliding in the ease of a built-up beam and therefore 12 STRENGTH OF MATERIALS the direction of the shearing stresses over the neutral plane in the case of a solid beam. ‘The above discussion shows that the shearing stress ro. at any point of the vertical cross section is vertical in direction and numerically equal to the horizontal shearing stress ray in the horizontal plane through “ thesame point. ‘This latter role stress can easily be caleu- lated from the condition of ee equilibrium of the element pom, cat out from the bbeam by two adjacent cross sections my and myx and by the horizontal plane pp, Fig. 109(a) and (8). The only forceson a @ tn. this element in the direction of the * axis are the shearing stresses ray over the side pps and the normal stress ¢= over the sides pm and py, If the bending moments at cross sections ‘mn and my are equal, ie, in the case of pare bending, the normal stresses o, over the sides np and mips will be also equal and will be in balance between themselves. Then the shearing stress ray must be equal to 280. Let us consider now the more general case of varying bending moment, denoting by Mand M + dM the moments in the cross sections mn and min: respectively. Then the normal force acting on an elemental area dd of the side npn STRESSES IN LOADED BEAMS 13 will be (ea. 57) My ty odd = Maa. ‘The sum of all these forces distributed over the side mppn of Re domet al [Mean In the same manner the sum of the normal forces acting on the side mpypon is [Bas a ‘The force due to the shearing stresses ray acting on the top side pp; of the element is tavbde. © “Theforcesgivenin (a), () and) must satisfy oq, 2X'~oyhence sebte = O NOY ay Ma, from which 5 = aM pt ta BS ta, or by using e4. (50), eee fae 6 ‘The integral in this equation has 2 very simple meaning. Te represents the moment of the shaded portion of the cross section, Fig. 100 (6), with respect to the neutral axis 2. Por the rectangular section discussed da = by and che itera becomes [oe -[EE-2(E-») @ ‘The same resule can be obtained by mulkiplying the area ns STRENGTH OF MATERIALS ‘(Al2) — ys] of the shaded portion by the distance 41(/2) + 911 ofits centroid from the neutral ais Substituting (d) in eq. (64), we obtain for the rectangular vee tom at (F-98) 6) Te is seen that the shearing stresses ue ate not unifrmly distribated from top to bottom of the Beam. The maximum wale of ru occurs for yx = 0, ey, for points onthe neutral xj and from equation (69), or, since I, = d#/13, 66) ‘Thus the maximum shearing stress in the ease ofa rectangular cross section is so per cent greater than the average shearing seen obtained by dividing the shearing force bythe area of For the bottom and for the top of the cross section, Ju = se hla and equation (65) gives rm, = 0. The graph of ‘equation (65) (Fig. 109, ¢) shows that the distribution of the shearing stresses along the depth of the beam follows a para- boliclaw. Theshaded area bounded by the parabola multiplied with the width 5 ofthe beam gives #(rpelauhé = Vas it should. ‘A natural consequence of these shear ing stresses is shearing strain, which causes eross sections, initially plane, t0 become warped. This warping can be easily demonstrated by bending with a force on the end a rectangular piece of rubber (Fig. 101), fon whose sides vertial lines have been drawn. The lines will not remain straight as indicated by the dotted lines, bbut become curved, so that the maximum shear strain ‘occurs at the neutral surface. At the points m’, m’, n,m! STRESSES IN LOADED BEAMS us the shearing strain is zero, so that the curves m'n" and mn are normal t0 the upper and lower surfaces of the bar after bending. At the neutral surface the angles hetween the tan- gents to the curves mn’ and mn and the normal sections san and myn, are equal 107 = 1/G-(rylonae As long a8 the shearing force remains constant along the beam, the warping of all cross sections is the same, so that mm! = mim’ 2m ‘= my! and the stretching or the shrinking produced by the bending momentin the longitudinal fibers is unaffected. ‘This fact explains the validity here of eq. (57), which was devel- ‘oped for pure bending and based on the asiumption that ‘ross setions of a bar remain plane during bending. ‘A more elaborate investigation of the problem ‘shows that the warping of cross sections also does not substantially affect the strain in longitudinal fibers if a distribueed load acts on the beam and the shearing force varies continuously along the beam. In the case of concentrated loads the stress distribution near the tf an cated, butthisdenia. 4 4 tion from the straight + T oo type (we Part I), : i loads P acting on the TF [> wooden rectangular f. Sqoiny om 5 fa Solution. ‘Thebending moment and shearing force diagrams are sven in Fig. 102, "See W. Voigt, Gottingen Abbandlangen, BE ge (EDs JA Michal, Quart Jf Math Val. 32 (gots and L. Ne G. Flo Ph Tr. Soc (eA, Va 21 God, and Kanon Roy. Sx Prt raga ovo 6 STRENGTH OF MATERIALS i Paw Ps Maa = Po rom egutons % P Bac and 32a, we obtain Pe sgj0lbs. and P= 10,700 Ibs; ‘therefore P = 5930s. is the limiting value of the load P. Fy picnic ies! i plea (ednee = 720 Ths. per 59:55 Cyan bp gn 4 Determine the maxim shearing ses inthe netal pane of sartomly loaded rectangular beam i the length of the fos 1 Get he lal per fot = tooo Isa the pth of the See sssion f= in and he with 32 Bn ae = 56.3 Ibs. pers im ave stmine the maximum shearing sues in problem 2 of 437. Distribution of Shearing Stresses inthe Case of « Circular Great Seon “InCncng the dhon ver © crear itvno foundation for‘the as. sumption thatthe. shearing sereses are all parallel to the SS SV shearing’ force. In. fact yd PA we can readily show that at Ie pointe p (Fig. toy, 2) of the a cross setion long the bounds” 7M Iy ary. the shearing sre is fern fhe dry. us consider an infinitesimal clement fed (Fig. 104, <) in the form of a rectangular paralllepied. with the face adn the surface of the beam an th face ad In the plane ak cross section, If the shearing sre acing over the seabed of the element has a direction such ae 7 it can alwaye be resolved STRESSES IN LOADED BEAMS a7 to two compet iin a radial direction and rein the direction af the tangent to the Bouadary. Now it has been proved before fee pe 111), By using the eustion of equilibrium ofan element that fa shearing stress atts over an elemental area, an equal SBeasing stress wil act also over an elemental ares perpendiclar tore. Applyng ths in our case it must be concladed that ia Sing stews acting on the clement aed in rail direction there must be equal shearing stress" on the side ay of the Clement fying inthe surface of th beam Te the lateral surface of the beni ee from shearing stresses the radial component of the sheng sees s must Be equal to ery hey mnt be in the Aitection ofthe tangent 9 the boundary ofthe cose section of the fen, "At the migpsine n ofthe chord pp symmetry ries that the shearing stress have the dinetion of the shearing force P: ‘Then the dtetions ofthe shearing strentes athe ponte p and wl intersect at some point Jon they axis (Fig. tog )- Asmming now that the shearing stress at any sther pala othe line pp ales, lrcted coward the pone 0, we havea complete determtation of {he directions ofthe shearing steaes As Second assumption We fae the vertial components ofthe shea {ng strees equal forall pints of te ine ppe Ae the assumption eeicides om letly with that made in the ate of @ {ectangular cows section, We ea We ef (Gg) forealelatingchiscomponent. Know Jing the actual direction ofthe she Shes snd te vetial component is tag nitude may be easly caleulated for any Doin of the cross seton Tet clot now the sheng ree lang the ine pp of the eras seston (Fig. 109) In applying eq (64) to the calefation Of the verial component yf these srcased we mst find the moment ofthe segment of he cle below the line pp with respect to the taxis ‘The lemental area mm has the length 2NIE ~ 9? tnd the width j. "The area is dd = aN ~ yy. The moment af tis sip thous Cea pdt andthe total moment forthe entre segment it "The appronimat cory based onthe above two aumptions giver satiatotony secuacy tnd comparison nth he eace theory shows that {he error nthe ragoitude of the metimom shearing str about § per cnt, which int high fr practi! apliaton. See Saint Venant, Nee Gey ptr. Se alto the book by Ay EH. Love, "Mathematial Theory of Barty” ah ed 1937, Bae 18 STRENGTH OF MATERIALS [xR Foey = Aue = yep, Substituting this in og (64) and taking 20% for b, we obtain te G and the total shearing stress at points p (Fig. 105) is a PRN 3 "Tey Me Te i seen that the maximum + is obtained for y) = 0, ie forthe neutral ax ofthe cross section. "Then, substting fy = FRey aa Tena Ip the cae of cialis co sto, hie the maximum sharing stress} per cen larger than the average value abened ty diving the searing fare by the womectionl sre 28. Distribution of Shearing Stresses in 1 Beams.—In yy considering the distribution of the shear- ing streses in I beams (Fig. 106) forthe section of the web, the stme assumptions are made as for a rectangular cross sections these were that the shearing stresies are parallel to the shearing force ¥ and ate uni- formly distributed over the thickness bof the web. Then eq. (64) will be used for calculating the stresses tar For points on the line pp at a distance j from the new tral axis, where the width of the cross sc tion is bi, the moment of the shaded portion with respect to the neutral axis +i Pont (f 8) 48-29) Substituting in eq. (64), we obtain FM) MH) (6s) Pra 1 OTH STRESSES IN LOADED BEAMS 19 ‘The stress then varies along the depth of the beam following f parabolic law. ‘The maximum and minimum values of tye in the web of the beam are obtained by putting y = © and joa hise tle -Fe-4]: (odun= oF ~ BE) oo ‘When dis very small in comparison with 5 there is no great difference between (7y)ase and (Fy:)ate and the distribution of the shearing stresses over the cross section of che web is practically uniform. ‘A good approximation for (ry2)au is obtained by dividing. the complete shearing force hy the cross-sectional area of the web alone. This follows from the fact that the shearing stresses distributed over the cross section of the web yield a force which is nearly equal to /, which means that the web takes nearly all the shearing force, and the flanges have only ‘secondary part in ics transmission. To prove this the ‘summation of the stresses r,- over the web which we will call 1p Praha fm 5) aro G4) +E ander inepton, ¥ [Moa ad bth hy (toe se ne © For small thickness of anges, ie when Jy approaches f, the moment of inertia, is represented with suficient accuracy GAM hoe ® in which the frst term represents the cross-sectional area of 0 STRENGTH OF MATERIALS the flanges multiplied by the square of the distance (b + A/ cof their eencers from the 2 axis, which is approximately the ‘moment of inertia of the cross section of the anges. The second term is the moment of inertia of the eross section of the web. Comparing (a) and (2), we sce that as hs approaches Ji the force V7, approaches and the shearing force will be taken by the web alone In considering the distribution of che shearing stresses lover the cross sections of the anges the assumption of no variation along the with of the section cannot be made. For example, at the level ae (Fig. 106), along the lower boundary of the flange, ae and de, the shearing stress tu, must be zero since the corresponding equal strest ta in the free bottom surface of the fange is zero (see p. 116 and also Fig. tog, ¢). In the part ed, however, the shearing stresses are not zero, but have the magnitudes calculated above for Gwaia in the web. This indicates that at the junction ed of the web and the ange the distribution of shearing stresses, follows a more complicated law than can be investigated by four elementary analysis. In order to dissipate a stress con- centration at the points ¢ and d, the sharp corners are usually replaced by fillets as indicated in the figure by dotted lines. A more detailed discussion of the distribution of shearing stresses in flanges will be given later (see Part ID. Problems 1. Determine (2a ad (rani the em ‘of an I beam, Fig 126,12 = § i By section of the webs Fin, A= a TTGENT esha tol imy = 30000 the, Determine the |] Satis ec tmtenited byte nb Pi 5 nae Jan = 6870 lb. Pe 3 (a = 4age be peregrina = ange 2 ese th ox her ages in he web ofa T beam (ig 109) b= 8 thy be 7 ins beet b= ging ds ~ rin. and V = 1,000 Ibs. if reson ner Using the sane mtd asin the xs of an fbeam, we nd (hon = 195 Ih. pe 9m 4 Determine the maximum shearing wre In pobiche 7 and 6 of article 35 STRESSES IN LOADED BEAMS m1 29, Principal Stresses in Bending.—By using eqs. (57) and (64) the normal stress ¢- and the shearing stress rw ca easily be calculated for any point of a cross section provided the bending moment Mand the shearing force J” are known for this cross section. The maximum numerical value of o, will be in the fiber most remote from the neutral axis, and the maximum value of rye usually at che neutral axis, In the majoriy of cases only the maximum values of os and tye ‘obtained in this manner are used in design and the cross. sectional dimensions of beams are taken so as to satisfy the ee and (ry nan oe It is assumed here that the material is equally strong in tension and compression and ais the same for both. Other fp yp ie SS z ei, Fo. tt wise the conditions of strength in tension and in compression must be satisfied separately and we obtain (obnne tim tension; (62) mia in compression. m STRENGTH OF MATERIALS There are cases, however, which require a more detailed analysis of stress conditions. We shall now demonstrate the ‘method of analysis necessary for such cases with a beam simply supported and loaded at the middle (Fig. 108). For « point 4 below the neutral axis in the eross section mm, the magni- tudes of the stresses ey and ryy = tay are given by eqs. (57) and (64). Tn Fig. 108 (6) those stresses are shown acting ‘on an infinitesimal element cut out of the beam at the point their senses being easily determined from those of M and V. For such an infinitesimal element the changes in stresses es and rye for various points of the element ean be neglected and it can be assumed that the element is in 2 homogeneous state of sress, ise. the quantities oe and tas may be regarded as the same throughout the element. Such a state of stress is illustrated by the element of finite dimensions in Fig. 373. From our previous investigation (see p. 46) we know that the stresses on the sides of an element cut out from a stressed body vary with the directions of these sides and that itis possible to so rotate the clement that only normal stresses are present (see p. 47). The directions of the sides are then called principal directions and the corresponding stresses principal sreses. ‘The magnitudes of these stresses can be found from eqs. (31) and (32) by substituting in these ‘equations 2, =o, ‘Then we obtain Te should be noted that ¢ngs is always tension and ¢, compression. Knowing “principal stresses, the shearing stress at any point will be obtained from eq. (34) (see p49) repent co For determining the directions of principal stresses Moht’s circle will be used. For an element such as at point (Fig. STRESSES IN LOADED BEAMS ns 108, J), the corresponding Mohe's circle is shown in Fig. 108 (©. By taking the distance OF = a, and DF = tw, the point D, representing stresses over the sides de and” ad of the element, is obtained. The distance OF is taken in the direction of positive ¢ and DF'in the upward direction because tes tensile stress and shearing stresses rye over sides be and fad give a clockwise couple (see p. 38). Point D, represents the stresses over the sides ab and de of the element on which the normal stresses are zero and the shearing stresses are negative, The circle constructed on the diameter DD, deter- i 14 and éaix = — OB. From the same con- struction the angle 2p is determined and the direction of ‘our in Fig. 108 (8) is obtained by measuring from the x axis in the clockwise direction. OF course emia is perpen- dicular t0 Gnas: By taking a section myn to the right of the load P (Fig. 108, a) and considering a point 4 above the neutral axis, the direction of the stresses acting on an element abcd at 1 will be that indicated in Fig. 108 (d). ‘The corresponding Moke’s circle is shown in Fig. 108 (@). Point D represents the stresses for the sides ab and de of the element abcd and poine Ds the stresses over the sides ad and Je. The angle y fetermining the direction ams. must be measured in the clock- wise direction from the outer normal to the side 0b or ed as shown in Fig. 108 (d). Tf we take a point at the neutral surface, then 2, becomes, zero, An element at this poine will be in'the condition of pute shear. The directions of the principal stresses will be At 4s" to the w and y axes. ‘eis possible to construct two systems of orthogonal curves whose tangents at each point are in the directions of the principal stresses at this point. Such curves are called the trajectories ofthe stresses, Figure 109 shows the stress trajec- tories for @ rectangular cantilever beam, loaded at the end, All these curves intersect the neutral surface at 45° and have horizontal or vertical tangents at points where the shearing stress tis Zero, ie, at the top and at the bottom surfaces of the beam, ‘The trajectories giving the directions of em we STRENGTH OF MATERIALS (ension) are represented by full lines and the other system of trajectories by dotted lines. Figure 110 gives the tra, jectories and the stress distribution diagrams for es and re ‘over several cross sections of a simply supported rectangular Eee “Ae Beam under wnifor lon tis clearly sean tha es hat ‘onimum vale at te mil wh te ening moment, isa maximum, and ry fa maximum at che support, where the maximum shearing force ats Th the dong of beams the concen is fr the numerically maximum flues of From eq. (72) ican be sen that fr the most remote bers in tension, where the shear is zeroy the longitudinal normal sess ou becomes the principal rs, ey oyu (ee For fer near eo the neutral aie the logiadinl Sher scress ons lens than at the extreme Ser however We now Bae ear ste ai ond these dr stg together at ths oine may proces prinpal sre ven by en (73), which wil be umerialy larger even than thet a the exereme Ber. In the ete of beams of rectangular oF Sircular erst section, in which the searing sees re ates continu down the depth ofthe beam, this nt ually the case, that iby the str (nue cleat fr the moet, remote aber atthe section of raninum bending moment is the maximum stress acting inthe bonn, However t Sch 2 case as an T beam, where sudden change cceurs in the Imagnitude of shearing sree atthe Junction of ange and tre (re p25), the anima stress calolated tt ont ef sree ae Vers 100 STRESSES IN LOADED BEAMS ns fot he bs oe las ts oe Gat aay nant section and the same dimen- T ee neue ee See ae yo00 X12 x6 (02a = BEE XG 7550 Ibs. per. Vou = We Now fora poine ae the junction of fange and web we obtain the fllowing values of normal and shearing stresses: = PEK ION 6610 ba. pera. ns Foe = 4430 Ibs. per ‘Then, from eq. (73), the principal stres is 8,830 Ibs. per sa in. Te can be seen that cam at the joint between the fange and the web is larger than the tensile stress at the most remote fiber and therefore it must be considered in design. ‘The ions of my tam Poa and Suis along the depth of the beam are showa in Fig. 11 Problems 1, Determine cage and vais ata point 2 in. below the neutral si in the soction 9 fet from the loaded end ofthe cantilever (Fig. fog) if the depth b= fin, the wideh & = 4 ine and P = 2/00 Ibs Determine the angle between cone at this poine and the x axis Salton, (e.) ~~ Sag Toes per a4. ims Cp) = 709 Tbe. pet sgn ee = $7 Hp ag In en = Se Th ‘The angle Between eas snd the x aie 8°16" measured clockwise ‘2 Determine suas and in at the negra avis and inthe cross section 1 foot from the left support for the uniformly loaded ns STRENGTH OF MATERIALS stant eam suport at the ends (ig 19). The es mensions are the same as in the previous problem, 000 Ib. per foots = 10 fet. ae 87.5 Ibs, per 39 in 13 Determine the length of the Theam considered on p. 135 if mes equal £0 cyae atthe junction of ange and web, Gaswor 1 = 398m 430. Stresses in Built-up Beams.—In engineering practice built-up beams are frequently used and the stresses in them ‘are usually calculated on the assumption that their parts are rigidly connected. The computation will then involve (a) the designing ofthe beam asa solid beam and (8) the designing land spacing of the clements which unite the parts of the beam. In the first case the formulas for solid beams are used, making an allowance for the effect of rivet holes, bolts, slots, etc by the use of reduced sections. The computations necessary for the uniting of the elements will be indicated by illustrations. Let us discuss fist a wooden beam built up as shown in Fig. 99. Itis assumed that the keys used between the two parts of the beam are strong enough to resist the shearing forces S (Fig. 99, 8). Then eq. (57) can be used for caleu- lating es. In order to take into account the weskening of the section by the keyways and the bolt holes, only the shaded portion of the section, indicated in Fig. 99 (@), should be taken into consideration. Then o-o [em ~ 07 In calculating the shearing force S acting on each key we assume that this force is equal to the shearing force dis- tributed in a solid beam over the area eb ofthe neutral surface ‘where bis the wideh of the beam and ¢ isthe distance between the middle points of the keys (see Fig. 99, a). Then by using eq. (66) and considering chat the depth of the beam STRESSES IN LOADED BEAMS 27 is equal to 24 in this ease, we obtain pode oo Tenens of he ys nd he teense DAs ONT fe ams sot a Sora ty sod cng fe nnd inc Edit stie opr Inve ts sant Ra dated owe mic een SESE ES Tal dette prone one tl ise tll ued he wer e58 Stating re Meenng tenes ee ur Ui in he Kg Eating stn Bens cis necessary to insure also suficent strength against shear- ing of the Wood of the beam along the fibers between two Keys. The shearing force will be again equal to Sand the resiating area is b X (¢ — a). Denoting with r.’ the working. stress in shear of the material of the beam along the fibers, the condition of strength becomes S_ say, Keone In addition to keys these are bolts (Fig. 99) uniting the parts of the beam. By tightening them frietion between the parts of the beam is produced. This friction is usually reglected in calculations and it is assumed that the total shearing force is taken by keys. Experiments show that such buile-up wooden beams are wesker than solid beams of the same dimensions.? "Phe experiments made by Proh E, Kiwll che Michigan allege ‘of Mines show that ballt-ap wen beams have sbout 75 pe cent of the Strength of the wld beam ofthe sme dimensions 128 "STRENGTH OF MATERIALS In calculating the oy strestes in built-up I beams the ‘weakening effect of rivet holes is usually taken into account by assuming that all the holes are in the same cross section (ig. 112, 2) of the beam * and subtracting their diametral sections in ealculating J, in ea. (57). Prana In caleulating the maximum shearing stress ru it is also the practice to take into account the weakening effect of the rivet holes. Tt can be seen that the cross-sectional area of the web is diminished, by holes, inthe ratio (e ~ de, where «is the distance between the centers of the holes and d the diameter of the holes. Hence the factor el(e ~ d) is usually included in’ the right side of eq. (64) for caleulating tye in the web of built-up I beams. Te should be noted that this ‘manner of calculating the weakening effect of rivet hoes is conly a rough approximation. ‘The actual distribution of stresses near the hoes is very complicated. Some discussion of stress concentration near the edge of a hole will be given later (see Part TT). In calculating the shearing force acting on one tivet, such as rivet 4 (Big. 112, 5), lee us consider the two cross sections 1mm and mins. Due to the difference of bending moments in these two cross sections the normal stresses 2» an sections smn and myn; will be different and there is a tendency’for the The holes in vertical web are prone im secns where vera silfeners are tvted tothe godess STRESSES IN LOADED BEAMS 1 flange of the beam shaded in Fig. 112 (0) to slide along the web. This sliding is prevented by friction forces and by the vet 4. Neglecting friction, the force acting on the rivet becomes equal to the difference of forces acting in sections rn and min of the flnge. The force in the flange in the cross section mn is (see eq. (2), 113) Hf vaa, where the integration should be extended over the shaded Cross sectional area of the lange. In the same manner for the cross section mn, we obtain ar ano (880 Pay “Thon the ce wanted by the vet rom the Aan rie web wl ae 528M fous. © By using eq. (50) and substicuting the distance ¢ between the rivets instead of ds, we obtain AM = Ve, where “is the shearing force in the eross section of the beam through the rivet . Substituting in eq. (a), we obtain fra. 6) ‘The integral entering in this equation represents the moment of the shaded cross section (Fig. 112, ¢) of the flange with respect to the neutral axis 2 Te is easy to see that in order to get sliding of the flange along the web the rivet must be sheared through two cross sections. Assuming that the force Sis uniformly distributed lover these two cross scctions, the shearing stress in the rivet 130 STRENGTH OF MATERIALS will be aay apy, J 744. 7) 7 ‘The force $ sometimes produces considerable shearing stress in the web of the beam along the plane ab (see Fig. 112, 8) which must be taken into consideration. Assuming that these stresses are uniformly distributed and dividing S by the area Be ~ d), we obtain v= Berta fue 0 In ado w this sre proce’ by fret Seamed fnmhe ge he il elon ese pac ee ing omens tae nding af he eb ie aces oF Ss scsi be aed by ae he shoe) Sid saben forte intl pd he ata aac Singer whe nur is sofecpoionst nee bins sr of the ne above te ane re Sanna we are ace flowing canton he oh trawnsin de web aingahe pone r in which th integrals extended ove shaded arn ofthe oo teealciersla ene: eae tao She oc and ser fr the Pins isthe se ob ca bo culated from eqs. (72) and (74), as was explained in the fren sean the dreds of pace en ok waoerae Trem the shove dicuson ti en that in eleating sets brop fem sera cpt eae oe Sling the cautions, ‘The con ease onan iitaatny fhe eee sevccs Shak fe ea conc chong the mekng seco bey ict tye brs ~FExpecinenes show that the fallre oF 1 beamg wpally azar dae te buckling ofthe compressed Manges or ofthe meb (eee Hl Fs Moore, STRESSES IN LOADED BEAMS ry Problems 1. A built-up wooden beam (Fig. 99) consists of two bars of rectangular cross section connected by eye. Determine. the ‘hearing free acting on the keys, the shearing stress inthe key and Dressure per unic area on its lateral sides if the load P = 000 Ibe, the width of the beam 8 = sin, the depth 2b = 16 in the wideh ‘of the Key a= 3 im the depth of the key 20 = 2 thoy and the Alsace beeweei centers of the keys ¢ = 4 in “aren 03 BEA aot, Shearing stress in the key is cB a tte prog 2S ata rg The pes eit hear ide po EZ ast erin 2. Determine the shearing srs athe neural ase of a girder, the web of which im thik and goin high the flanges cone fon put of angles Gin. Bin han when the fra sheating foreon the scons 1gopee Ihe Detzemine ls the shearing stress the ves atachng the anges to the web ifthe dametee AF thee fet itin and the pitch = 4 in p12 Snuion. For the deasions given we have Bx + ating + 595 x 2990 The moment of half ofthe cross section with respect tthe neta 25x35 [sd PE 2 595 259 son int In this calculation £75 in? isthe cross sectional area of an ange, 1g 9s the moment fetal the rey section ofan angle ack University of Minos, Bullen 6, 919). Te quention of buckling wi beconidred later. "Thee of bending af svets onthe otbuten of straes in I beame has tees dicuned by Te Arnovlersy Zatch f Architekt, w. Ingericurwesen, 1910 ps7.” He found that due to this tending stress for uate proporcion increase about 8 per ont 132 STRENGTH OF MATERIALS peers sree peer eFicam, Shs in ebettocce el eh2cequcl each ani fom ee eee ae tam eden: Ales asec sso; X 08 x 20400 we consider weakeningof che web by the rivet holes, chen (mee = 4,20 Ibs. per agi (rade 4 4920 © 6,560 Ibs per sq. ine 3 ‘The force $tranamitted by one rivet, from eg. (96) go BOOKA KAM se cr ‘The shearing sree in the rive, from eq (7)s plo? a 1 BEX? = sono Ih. por 0g 13 Determine ena in points ofthe plane ab (Fig. t12) a distance ‘of 215 in fom the neutral axis ifthe dimensions ofthe bes the rame asin the previous problem, = tSoja0 and the bending moment M = 3 X10 Tbs. in ‘olution. From eq. (78), 4920 15050 ~Teae90s 3 gx 1 x ang Vite eee eee Pepe ees pe neha epee! Sry P ae 2 acme git Pc teere ante cer eee Sea cake an a ee His tetany ns te cee ra aehace’ Chen i ren eine (268 + 61) = 4goolbs. per sq. 1460 Ibe. er 9 im : 5360 Ths. pers in Pr. 1 Solution STRESSES IN LOADED BEAMS se 000 x 6X 99 Aye ioxs)” 2,790 Ibe, 13 CHAPTER V DEFLECTION OF TRANSVERSALLY LOADED BEAMS 3%. Differential Equation of the Deflection Curve.—In the design of a beam the engineer is usually interested not only in the stresses produced by the loads acting but also in the deflections produced by these loads. In many cases, furthermore, it is specified that the maximum deflection shall not exceed a certain small portion of the span. Prot Let the curve AmB in Fig. 114 represent the shape of the axis of the beam after bending. This curve is called th defection curce. To derive the differential equation of this ‘curve we take the coordinate axes as shown in the figure and assume that the curvature of the deflection curve at any poine ‘depends only on the magnitude of the bending moment M at that point.!” In such a case the relation between the curva. ‘ure and the momentis the same a in the ease of pure bending. (Gee equation (56)), and we obeain @ {The fet of shonrng force om dhe curvature wll be dacued ae (ace aro). Tew be shown that tis efecto woully awa aad ae Benesected. DEFLECTION OF LOADED BEAMS Bs ‘To derive an expression for the relation between the curvature and the shape of the curve, we shall consider two adjacent points m and mi, ds apart on the deflection curve. If the langle which the tangent at m makes with the x axis is denoted by 8, che angle between the normals to the curve at m and ‘m, is do. ‘The intersection point O of these normals. gives the center of curvature and defines the length r of the radius of the curvature. Then dsm rdt and E > a . ® the bars indicating that we consider here only the numerical value of the curvature. Regarding the sign, it should be noted that the bending moment is taken positive in equation (@) if it produces upward concavity (see p. 71). Hence the curvature is positive when the center of curvature is above the curve as in Fig. 114. However, it is easy to see that for such & curvature the angle 8 decreases as the point m moves along the curve from 4 to B. Hence, toa positive increment 4s corresponds 2 negative a, Thus to have the proper sign ‘equation (6) must be written in the form In practical applications only very small deflections of beams are allowable, and the deflection curves are very fat. In such cases we can assume with sufficient accuracy that ds mds and 0 = tan = dylds. @ ee tGecsbieh os © Equation (a) thus becomes EL ) 136 STRENGTH OF MATERIALS ‘This isthe diferendal equation of the deletion curve which muse be integrated in cath particular ease to nd defections of beams. Te should be noted that the signin equation (79) depends spon the direction of the coordinate axes. For example, if wwe take y positive upwards, i is necesary to pat 0 = ~ dylde in place of equation (4); and we obtain plus instead of minus on the right side of equation (79). In the case of very slender bars, in which the deflection may be large, it is not permissible to use the simplifications (@)and we must have recourse to the exact expression won (f) sen a ay a ay [+] Comparing this result with equation (cit can be conchuded that the simplifications shown in equation (2) are equivalent to assuming that the quantity (dyids}in the denominator of the exact formula (f) is small in comparison with unity and can therefore be neglected * By differentiacing equation (79) with respect to «and sing equations (so) and (51), we obtain she 0 The exact cxpreation f) for he carvatare war wed by whe Et investigators of the defection curves. Te watson for example, by E"Bhler in his aris work on "Elastic Curren” an Engh eanltion of which was published in "Ios," No. s8 (Vol XX, 1) November, 1933. DEFLECTION OF LOADED BEANS 7 and “ a8 (0) ‘The lat equation is sometimes ted in considering the de flection of beams under a distributed load, 32, Bending of a Uniformly Loaded Beam.—In the case of a simply supported beam which is uniformly loaded, Fig. 63, the bending moment at a cross section mm, a distance x from the left support, is and the differential equation (75) becomes ay ge gt agate Multiplying both sides by di and integrating, we ob 4 ato a G= Be © where Cis the constant of integration which ist be adjusted to satisfy the conditions of this partcular problem. To this fend, we note that as a result of symmetry the slope at the middle of the span is zero.” Setting dyids = 0 when = swe thus obtain -£ . a and equation (a) becomes ‘ ELE = - 7 ® A sonnet give ‘The new constant of integration C, is determined from the condition that the deflection at the supports is zero. Sub- 138 STRENGTH OF MATERIALS stituting y = 0 and x = 0 into equation (¢) we find C; Equation (6) then becomes ya aby. tte ae +99 en ‘This is the defection curve of a simply supported and uni- formly loaded beam. The maximum deflection of this beam is evidently ae the middle of the span. Substituting x = 1/2 in equation (81) we thus find -ig Jou = 3g ET, @ ‘The maximum slope occurs at the left end of the beam where, ‘by substituting x = 0 in equation "Ty, @rweobsain comm us ab) = 2)..7 ain 9) nett! In the case of a sniformy Jaded cantilecer beam, Fig. 1154, AIM + the bending moment at a cross “ section mn a distance x from the lefeend is ly un, etl ait ‘The first integration gives dy ae aZ- Bese, @ ‘The constant of integration is found from the condition that the slope at the builtin end is zero, that is dyjde = 0 for x =. Substituting these values in equation (a) we find e ce-¥. DEFLECTION OF LOADED BEAMS 139 ‘The second integration gives sgh ae Ely = © +6. w ‘The constant C\is found from the condition that the deflection vanishes at the builtin end. Thus, by substituting x = 4, 9 = on equation (6), we obtain af Substituting this value in equation (2), we find 4 y= gdp (t-te tah 4) By ‘This equation defines the deflection curve of the uniformly loaded cantilever. If the left end, instead of the right end, is bule in, asin Fig. 1155, the deflection curve is evidently obeained by sub- stituting / — x instead of x in equation (84). In this way we find Ltt h Joy (xt = a + 6). (3) Problems simply supported and uniformly loaded wooden beam of quate com section has a span / = to fe Find the maximum ‘Seflcction if (eaux = 1200 Ibs per sq iy B= 1.5 X 10" Ibs per Sins and g ~ 420 lbs perf, 12 Rind the depeh of = uniformly loaded and simply supported steel I beam having span of 10 fe the maximum bending stress E600 Ibe. per sq. in. and the maximum deflection 3 = 0.1 in newer: ‘b= 16 5A niformly forded cantilever beam of a span = ro fy hast defection atthe end equal to o.%/. What i the slope of the deflection curve at the end? Se, What fs the length of uniformly loaded cantilever beam if its defection atthe free end jsf. and the slope of the defection curve atthe same point is 01? 10 STRENGTH OF MATERIALS 5. A uniformly loaded stel I beam supported atthe ends ha deficction a the middle of 8 = s/s6 in. while the slope of the deflec- Soncarveat the tnd = dol Hinde depth hl he kam the Kiam bending set se Wels the knows formulas From the fat two formulas we find & i Sx too in and = toon. "The second formula then gives AED _ 3X 0X tt X01 an 10 . Substituting this in the third formula, we obtain a 2X thee X 100 FX PX Is Kook 33. Deflection of « Simply Supported Beam Loaded with Concentrated Load—In this cate there are ewo diferent py expressions forthe bende ing moment (seep. 75) +: corresponding to the two portions of che beam, Fig. 116. Equation (79) for the deflection curvesmust therefore be written for cach portion. In this way we obtain ee ‘ in, ay ELS =~ and wees Pea fe nae DEFLECTION OF LOADED BEAMS ut By integrating these equations we obtain dy Pht ee ed and © ¥ (Phx | Plx — a) si = — Py PERO g, for x= Size reo bai he defen core mt be Sa oe aden a te lod cp eget Fe he en be ea ee eds date come ot Co Sea ae ea ee Robraing me mond Er fie acdag Ch Ca wesboe my- sora t zee | and ® Bty = PH POEM oe fer aE Since the two branches of the deflection curve have a common Geflection at the point of application of the load, the two expressions (2) must be identical for x = a. From this it follows that C; = Cx Finally we need to determine only two constants C and Cs, for which determination we have two ‘conditions, namely that the deflection at each of the two ends of the beam is zero. Substituting x = 0 and y = 0 in the firs of expressions (8), we find Ge Gao © Sabstituting y = 0 and x =/ in the second of expressions @ we cba Pe Pe, © Substituting the values (@) and (d) of the constants into ‘equations (B) for the deflection curve, we obtain 2 STRENGTH OF MATERIALS Phe on MeOH Ba te wae 06 sty = eas PEM oe cau ‘The firs of these equations gives the deflections for the left Portion of the beam and the second gives the deflections for the righe portion. Substituting the value (d) into equations (a) we obtain 4D ee a er and © Bide Peep 30) 4 PERM From these equations the slope at any point of the deflection ‘curve can readily be calculated. Often we need the values of the slopes at the ends of the beam. Substituting ¥ = 0 in the first of equations (¢), x = Jin the second, and denoting the slopes at the corresponding ends by 0; and és we obtain? dy) _ PaO Bae a) ’ ote The maximum defection occurs at the point where the sangent to the deflection curve is horizontal. If'a'> bas in Fig. 116, the maximum defection is evidenely in the eft Porton of the eam. We can find the portion of thi pone by equating the first of the expressions (eto ero to obtain P- B38 wo, For flat curves, which we have in most cases, the slopes 0, and 0 be fakenmumerily gaa othe angles ef taton of Se ete of $8 ou ‘tg tending. he pes Bag talc wale hese a (9) DEFLECTION OF LOADED BEAMS us from which _ =e 7) a ‘This is the distance from the left support to the point of maximum deflection. To find the maximum deflection itself wwe substitute expression (f) in equation (86), which gives = a ET. If the load P is applied at the middle of che span the maximum defection is evidently at the middle also. Tes magnitude is obtained by substituting 5 = //2 in equation (g), which gives @ Oey = GET Co) From equation (/) i can be concluded that in che case of one concentrated force the maximum deflection is always near the mile of the bears. When # = // iis ae the middle: in the limiting case, when 4 is very small and P is at the suppor, the distance + as given by equation (/) is 3, and the point of maximum defection is only a distance ey hobs cont from the middle. Due to this fac che defection at the middle is a close approximation to the maximum defection, To fobtsin the deflection at the middle we substitute x = df in equation (86), which gives Odssye © goer a — 40. oo The dliference of the defections (g) and (91) in the most unfavorable case chat is when # approaches zero, is only about , ‘a5 per cent of the maximum deflection. Mt STRENGTH OF MATERIALS Problems 1. Find the position of the load P, Fig. 116, if the ratio of the numerical values of the slopes at the ends of the beam is 2 Find the difference between the maximum defection and the ‘deflection athe middle of the beam in Fig 116 if 9 — 3a 4. Find the maximum deflection ofthe beam shown in Fig. 116 if dB isan American Standard T beam, € in. in depth and $34 2 in rosesetonl ares, and ¢ Ha fey BN fey and P= ‘What willbe the maximum defection if the T beam of the previous problem is replaced by « wooden beam having cross section 10, by 10. The modulus of elasticity for wood can be taken as E = 115 X10 Iba, per afin. 44, Determination of Defections bythe use of the Bend- ing Moment Diagram Method of Superposition Inthe pre. ceding ails it was shown how the defection curve of « beam can be determined by incegration of the dierentil equation (9). Ta many eases, however, epoially if we need the defection in « prescribed point rather than ‘the. gen- tral equation of the deflection tures the calelaton ean. be considerably simplified by the tee of the bending: moment Glagram as willbe devribed in the flowing discussion! In Fig. 117 4B represents portion of deflection curve and. ash the corresponding portion of the bending moment af trans wes droped © Maye Blend Aes ingen Vercn on Hanover, pin See soso. has teen at Sen Gait dor Tcaacen Mecano Bee ook timer method war Sven indepndeny of. Mi 6) Pro Gren Une of Man he DEFLECTION OF LOADED BEAMS 4s diagram, Two adjacent cross sections of the beam at dis- tance ds apart will intersect after bending, at an angle do, and, from ea. (56), : ao mds = Beds For beams used in structures, the curvature is very small, land we may use ds for ds. ‘Then a = pp, (de). @ Graphically interpreted, this means that the elemental angle 1d between two consecutive radii or two consecutive tangents to the deflection curve equals the shaded elemental area Mas: of the bending moment diagram, divided by the flexural rigidity.® This being so for each element, the angle @ between, the tangents at f and B will be obtained by summarizing such elements as given by eq. (@)- Then oo fh emen wo the angle between the tangents at two points A and B ‘of the deflection curve equals the area ofthe bending moment tiagram beeween the corresponding verticals, divided by the flexural rigidity of che beam. ‘Let us consider now the distance of the point B from the tangent AB’ at point 4. Recalling that a deflection curveisa flat curve, the above distance can be measured along the vertical BB'. The contribution made to this distance by the bending of an element mn of the beam and included between ‘the two consecutive tangents at m and n is equal to Max Ee Interpreted graphically this is 1/1, (moment of shaded area ‘Mads with respect to the vertical through B). Integration 7 By way of dimensional checks a isin radians Le, «pare number, Marini Tb ince, Hi Tho etait Caches) xd = Ms STRENGTH OF MATERIALS ives the total defection BB: TE ~ ff ppt on that is, the distance of B from the tangent at is equal to ‘the moment with respect to the vertical through B of the area of the bending moment diagram between and B, divided by the flexural rigidity £Y,. By using eqs. (92) and (93) the slope of the-defection curve and the magnitude of deflec- tion at any cross section of the beam can easly be caleulated. ‘This method of calculating defections is called drea-Moment Method. bet Ieshould be noted that the deflection of a beam ofa given flexural rigidity (see equation (93) is entirely defined by the bending moment diagram. From this fact a very important conclusion can be drawn, It will be appreciated from the definition of the bending moment (art. 19) that che bending ‘moment produced at 2 cross section mn of a beam by several simultaneously acting loads is equal co the sum of the mo- ‘ments produced at the same eross section by the individual Toads acting separately. On the basis of this conclusion, together with equation (93), it can be stated that the deflec- tion produced at a poine of a beam by a system of simultane DEFLECTION OF LOADED BEAMS ur ‘ously acting loads can be obtained by summing up the de- Fections produced at that poine by the individual loads. For ‘example, if the deflection curve produced by a single load (equations (86) and (87)) is known, the deflection produced by several loads is obtained by simple summation, This method of calculating deflections will be called she method of superposition in the subsequent discussion. ‘The ealeulation of the integrals in equations (92) and (93) canoften be simplified by the use of known formulas concern jing areas and centroids. Several formulas which are often encountered in applications are given in Fig. 118, 13. Deflection of a Cantilever Beam by the Area-Moment ‘Method.—For the case ofa canti- lever beam with a concentrated load at the end (Fig. 119, a) the bending moment diagram isshown in Fig. 119 (6). Since a eangent atthe built-in end 4 remains xed, the distances of points of the de- flection curve from this tangent areactual deflections. Theanglet Which the eangent to the deflection ccurve at B makes with the tangent at 1 is called the angular defection of B with respect to 4. ‘Then from ea. (92) " ryt Pe ae PIX x gr = er Cc “The deflection 8s calculated from o9.(93) as the moment of the area aba; about the vertical through & divided by EZ Thea Ly 2yy he BP b= Pixlx2ix ge = Fr 9) or any eros section such as mm, the angular defection from The numeral value of the angular defection ix eaealated. The linccuon of the defection is readly seen fom the loading condition ws STRENGTH OF MATERIALS Ais the area m/n'aa, of Fig. 119 (0), divided by EI, In the ‘ease of flat curves such as deflection curves of beams, angular deflection can be taken equal to the slope of the curve and we obtain o_o de” ET, 1-5] i ‘The defection a the same cross section is che moment ofthe area m/nvaay about mn divided by El (see eq. 93). Separat- ing this area into the rectangle andthe eiange indicated in the figure, this is yegp[re- 92+ 22]- 2 (E-2): on For a cantilever with a concentrated load P at a cross sec- tion a distance ¢ from the support (Fig. 120, 2) the beading moment diagram is shown i Fig. 120 (#). The slope and the deflection for any section e 7° to the left of the pone of ap. plication ofthe load are deer- xy a mined from eqs. (66) and (67) SNC Bray oe {ros section tothe right ofthe load the bending moment and the curvature are zero; hence this portion of the beam re- mains straight. The slope is constant and equal to the slope at D, ive, from eq. (94), Pa/aEl,. The defection at any ‘ross section mn i the moment of the area ofthe triangle aad about the vertical m’n” divided by EZ., which gives 1 Pey, a oo ee (*-4) ) In the case ofa cantilever with a uiform load of intensity ¢ (Fig. tans) the bending moment 4 any eres section DEFLECTION OF LOADED BEAMS Ww sm distant #; from the builtin end is ‘The slope at any cross section a ance * from the support is, from ea. (92), ea Wan, ‘ ES ean ah. (%- 4+). on ‘The slope at the end is obtained by substituting / for x in the above equation, giving (2). a. t= ‘The defection at any section a distance x from the builtin end ie the moment of the area aayed about the vertical ed divided by EZ, (Fig. 121, 8). ‘The moment of the element of this area, shown shaded, is teen eee posed aes oe ceane ie Eni, © 00-9 ‘The deflection at any point a distance x from the support is then, after integration, (ro) 150 STRENGTH OF MATERIALS For the deflection at the end « = 10m fh ven ‘The same problem can be solved by using the method ‘of superposition. ‘The uniform load ean be considered as a system of infinitesimal loads qdc as indicated by the shaded area in Fig. 122. The deflection ime produced at the cross section mn weit} bretch elemental load gde to j—+ left-can be found from eq. (98) by > substituting ade for P. "The de- flection 7, produced by the total Fis na load to the left of ma is the sam- mation ofthe deflections produced by all such elemental loads with ¢ varying from ¢ = 0 to eB f€ t-te sft Te dafleson pda the se ton mo by a ee rent load py tts ght fund om eG 8p Subsitutinggdefor Pande for Thedfccion9! poriaced nn by the ttl oa tothe tights these he defeeons duc to all sch cement lind, wih or vag fomacewant wea L(S -~E)e = str(- ‘Then the fra deecton atthe section mix sonsnnihi (EE 8) which agrees with eq. (101) found above, Problems 1. Determine the defection and the slope of the cantilever beam in problem 9, p. 108. DEFLECTION OF LOADED BEAMS 151 Slain. ao Phy ge = 3ET, + eer, 2. Detersine the deletion ofthe rp ofthe pla represented in Fig. 9. ‘Slaton, The bending moment at any cros ection mn, & lseance from the op we u- -, whee 97 = 4a x 62.4 Ihe the otal hydrate presare tran Mite to one pillar. ‘Using ey. (a) the defection of the tp of the pila is He fii WE sx6x6r4 XOX 2X1 Blade gh 7 ISBT,” 2xIgKTSXIOXSSF 43, Determine the defection and the slope at the end of «cant lever bent by # couple M (Fig. 133) omnes (2) a? es t - ——! 4- Two wooden rectangular beams clamped at the left end: (ig 124) are bent by eightening the bole ae the right end. -Deter- rine the diameter dof the bolt to make the factors of safety for the wooden beams and for the ste! bolt the same. The length of the beams 73 fet, the depth A= 8 ina the width B= 6 iy working stres Yor steel wy = 12,000 Tbs. per sina for wood z= 1,20 Ib. pet sq in. Determine the deflection ofthe bears ‘when the tense stress inthe bolt fs 12;000 Ibs. per 2 ‘Solution. ICP is the force inthe bol, the equation fr deter mining the diameter d willbe 4P 6! _ 12900 24 BB > ra00 152 STRENGTH OF MATERIALS from which 130 Tbs & oa7sin, and P= t2po0 xf ‘Then from o4. (95), by taking E = 1.5 X 108 Ibs. per spin the Aefecton 3 = octby in eee §. What isthe rio ofthe defection atthe ends of the cant levers shown in Fig. 125 che intensity of uniform load i the me in'both eases? a ire 6, What must be ehe equation ofthe axis of the curved bar 4B before is benef the load P, moving slong the bar, remains always, fon the same level (Fig. 126)? Annee yf 2. Determine the safe defection ofthe beam shown in Fig. whch he woking te fe gven, Determine th aor & ‘lovee loaded Sethe tad (BET ‘fuer ot @s anit 8. A circular dice N of radius R (Fig. 17) inoue produces on 2 thin stecl strip of thickness A En acracton of fie par in, ily distributed. Determine the length / of the uncupported part ofthe stip andthe maximom sess in eit A= orice = 3 Jing and q= 1g Ibs. peraq in Soluton. ‘The length of the unsupported part of the strip can be determined from the condition that tt she pein C the curvacure produced by the uniformly distributed load'g must be equal to DEFLECTION OF LOADED BEAMS 153 UR, Therefore Eh Po Re from which ‘The maximam stress is determined from the eqution tau = EujaR = Sono Ib. per. 1, Deistnine the defections of the cantilever beams shown in Fig 6, asuming that the material i stel, the depth of each team ip to iny sod the maximum bending stress i 16,000 Tb. peroq in 36. Deflection of a Simply Supported Beam by the Area Moment Method—Let us consider the case of a simply supported beam with a load P applied at point F, Fig. 138. The bending moment diagram is the triangle dubif ‘Fon, Its aren is Pabla and its centroid C is at distance (U-+ 8)/3 from the vertical Bhy. ‘The distance 8 from the end B to the line 8’ which is tangent to the deflection curve at is ob- tained from equation (93) and is Lb _ Pabll +8) ‘ 37” 6El, E 1st STRENGTH OF MATERIALS By using this value the slope @ at the left end of the beam is found to be Pabll +b = 5 ~ Pee, @ which coincides with previously obtained formula (88).7 A. simple interpretation of the formula (a) is obtained if we consider ay; as a simply supported beam, carrying the tr angular load represented by the triangle aif. The reaction at the let support a of this imaginary beam is evidently Pab tb 1 Palit +) 2 XX a By comparing this with formula (a), it ean be concluded that the slope 4 is equal to the reaction at the corresponding support ofthe imaginary beam divided by the flexural rigidity ‘of the real beam. The slope @ at che right end of the beam ‘ean be obtained in a similar way; to get the correct sign for the reaction at the right end must be taken with minus sign, which represents the shearing force at the right end of the imaginary beam. The imaginary beam a,b, which carries the fictitious load represented by the area of the bending moment diagram is called a conjugate beam. It can thus be concluded that the numerical values ofthe slopes atthe ends of 2 simply. ‘supported beam can be obtained by dividing the reactions at the ends of the conjugate beam by the flexural rigidity 21. ‘This conclusion, which was derived for the case of a single load, holds for any transverse loading, since, as has been shown (p. 146) the moments and the defections in the case of several loads can be obtained by the superposition of the ‘moments and deflections due to single loads To calculate the slope at any point d of the deflection curve, Fig. 128, cs necessary to subtract the angle @ between, the tangents at 4 and at d from the angle é at the support. Using equation (92) for the calculation of the angle 8, we obtain R TT Nate tat DEFLECTION OF LOADED BEAMS 185 Bo gon ce sm. Tie fine sam in the pnts she raion hee upport of the conjugate beam ay and the second i the load See pace Wi owe ek ef he ao ea oe Fe eae prendschrereromns The ee cet ls utn mr of he enfogt ean cere te ho th ae beam att pot den Sa EB cing face ae epord ee ate oe ney ace Bhd Er * Conddering next che deflection y at a point dt may be ection fata dae yank ® From the tangle dee we obtain the relation wae © where R is the reaction at the left support of the conjugate beam, ‘The second term on the right side of equation (6) represents the distance of the point d of the deflection curve from the tangent de and is obtained from equation (93) as HE = py_ares Bann x5 @ Substituting expressions (e) and (d) in equation (8), we obtain > -( Re = Aan x?) fee eae Derreeninperat ee 156 STRENGTH OF MATERIALS Post area Aaymn = PA, Bas, Pabs(l+ 8) _ Post) Phe [Ae ar | oper “This checks with equation (86), which was previously ob- tsined by integration of the diferental equation of the de. ection curve. The defection fora pint to the right ofthe load P can be calculated in a similar manner, ‘The result wil of course, be the same as equation (7). Having the deflection carve produced by a singe oad P, the defection curve produced By any system of transverse concentrated loads can readily be obtained by employing the method of superposition. It is simply necessary to use equations (86) and (87) foreach individual lad, applicable tothe case of «div trluted lads an example F Simply supported beam under 4 uniformly. daributed Toad, ‘The same method is also oy a # wwe shall take the case of & Fig. 129, and calculate the “ slopes atthe ends and the de- hd flection at the middle, From o equation (a) the increment of Pa slope d¥, produced at the lfe end of the beam by the ele- ment of load gdb shown in the Fig. 129 is = wehdbl + 8) _ ght — Bb = oe ce ‘The slope 0, produced by the total load is then the summation of the increments of slope produced by all the elements gdb fromb=otob=l Thos BP — bab _ gh oo fn? = afr, v DEFLECTION OF LOADED BEAMS as ‘The deflection at the middle is obtained from equation (91), which was derived on the assumption that the load is to the Tighe of the middle. Any clement of load gdb to the right of the middle produces at the middle a deflection hdd an = Ser, ‘Summing up the deflections produced by all such elements of load to the right of the middle, and noting that the load on the left half of the beam produces the same deflection at the middle as the load on the right half, we obtain for the total deflection = 0 gbab 5 ane [Sh or = Ea © “The results (f) and (coincide wth formulas (83) and (82) previously obtained by integration ofthe diferent equation UF the deflection curve. “The same resalts ar readily obesined by considering the conjogate beam ah Fig. 29h loaded bythe parabolic segment Sth ich isthe bending moment diagram tn this case. ‘The {otal tition lal on the conjugate beam is aya xtxy and each reaction is equal to 4/24. ‘The slope () is then Sbesined by dividing this reaction by EZ. To calculate the ‘Tefecton atthe middle we find the bending moment at the mmiale dof the conjugate beam, which is a(t al) _ sat, “( %) oe “The defection (g) is then obtained by dividing this moment by Er : ‘The method of superposition ig f--¢—=-TTIITI speilly usefl if the distributed ee ford covers only & part of the span fin Fig, 170. Using the expression developed above for 158, STRENGTH OF MATERIALS (Benny the deflection produced at the middle by the load to the right of the middle is ts a= [Seon — ay The load tothe lft ofthe middle produces the deflection n- [ao-on wet in= [eon + [ Inthe ceo sinp spre em 48cm diene Sole aie Perec dee eer ea on eae in el el Bilt ttactecet aT he Soe ‘ iP ~ 46). at the ends of the actual beam are (103) and (104) ‘The sign of the slope at the righ end is of course negative. ‘The deflection at a cross section mo of the beam is obtained by dividing the bending moment at the corresponding. cross section mm of the conjugate beam by El,, which gives DEFLECTION OF LOADED BEAMS 189 ey (ee Bt) - GE (1 F)- 09) Problems 1. Determine the angles at the ends and the defection under the loads and at the mide ofthe beam shown B12 eon wn est thete “Slaton ‘The conjugate beam wil - boelonded by the trapeoid ade, the area 744 ofvwhichis Pec). Theangies at the ae z fends are i “Palo + m2 Pra 13 “The defection under the load i ee wag) enh ‘The defection at the midi, from eq (91) is Pe ayer ~ ane 4, Determine the slope a the ends ofthe eam shown in Fig. 88 nseer. 4) 1, (#) ~_ 8m de).7 ET ée)_.= ~ 18 ET, 1} Determine the defection at the middle of the beam 4B, shown in Fig. 133) hen Ze gains y= fl |p §20 Iba. pet foot, = 24 feta = 12 het, MUU] fA FAS fee Em 30% 10! Ih, pr a9. —: Solution. Due to the fact tht ¢ ‘= Jin" the defection prodiced. at the middle ‘by the load acting on the left half of ehe beam, from eq. (82), is ieee Oden LES “The defection prodvced at the middle by the load on the right

Anda mungkin juga menyukai